ekadashi 58 dyhannia-testy

http://zavantag.com/docs/2819/index-83894-8.html

Крок 1. Стоматологія. 2004 - 2005 рік

6.0 Патологічна фізіологія




Яка з перерахованих буферних систем слини не приймає участі в регуляції кислотно-основної рівноваги в порожнині рота?

@ Гемоглобінова.

Фосфатна.

Бікарбонатна.

Білкова.

Жодна.

#




На прийомі у стоматолога під час екстракції зуба у пацієнта раптово погіршився стан, внаслідок аспірації зруйнованих тканин зуба. Який вид патологічного дихання може вірогідніше за все виникнути у хворого?

@ Стенотичне.

Чейн-Стокса.

Біота.

Тахіпноє.

Кусмауля.

#




У хворого виявлена стійка гіперглікемія. Артеріальний тиск в межах норми. Який вид гіперглікемії має місце у даного хворого?

@ Гормональна (недостатність інсуліну).

Психоемоційна.

Аліментарна.

Гормональна (патологія кори наднирників).

Гормональна (патологія мозкового шару наднирників).

#




Одним із факторів утворення виразки шлунка і 12-палої кишки є підвищення кислотності шлункового соку. Який із гуморальних факторів відіграє вирішальну роль у механізмі гіперсекреції соляної кислоти?

@ Гастрин.

Серотонін.

ВІП (вазоактивний інтестинальний поліпептид).

Секретин.

Холецистокінін.

#




У хворого щелепно-лицьова травма, яка дозволяє прийом лише води протягом щонайменше тижня. Який вид голодування буде спостерігатися у хворого в цей період?

@ Повне голодування з вживанням води.

Абсолютне.

Часткове.

Неповне.

Якісне.

#




Хворий через наявність пародонтиту відмовляється від вживання продуктів харчування, які необхідні організму, але потребують пережовування. Який вид голодування може мати місце у хворого?

@ Якісне.

Повне.

Абсолютне.

Неповне.

Недоїдання.

#




З приводу злоякісного новоутворення верхньої щелепи хворому проводиться променева терапія. Наслідками прямої дії іонізуючого випромінювання будуть наступні структурні зміни в клітині, крім:

@ Збільшення кількості мітохондрій.

Розрив хромосом.

Розщеплення молекул ДНК.

Розщеплення молекул РНК.

Набухання органоїдів клітини.

#




Хворий уражений електричним струмом в ділянку жувального м'язу. Через який із перерахованих нижче органів проходження електричного струму є найнебезпечнішим?

@ Кора великого мозку.

Легені.

Нирки.

Печінка.

Слинні залози.

#




Хвора, 30 років, звернулася до лікаря-стоматолога з приводу спонтанного перелому правого верхнього різця. Із анамнезу відомо, що у хворої часті карієси. Два роки тому була проведена резекція правої долі щитоподібної залози. Яке порушення з боку ендокринної системи має місце в даному випадку?

@ Зниження утворення паратирину.

Підвищення утворення паратирину.

Зниження утворення тироксину.

Підвищення утворення тироксину.

Підвищення утворення кальцитоніну.

#




На прийомі у стоматолога під час екстракції зуба у пацієнта раптово погіршився стан, внаслідок аспірації зруйнованих тканин зуба. Який вид патологічного дихання може вірогідніше за все виникнути у хворого?

@ Стенотичне.

Чейн-Стокса.

Біота.

Тахіпноє.

Кусмауля.

#




Чоловік. віком 35 років, при роботі з джерелом іонізуючого випромінення отримав радіаційне ушкодження організму середнього ступеню. Яке порушення в системі крові буде спостерігатися у нього в першу чергу?

@ Лімфопенія.

Анемія.

Гранулоцитопенія.

Тромбоцитопенія.

Геморагічний синдром.

#




У дитини, віком 5 років, була виявлена аденома гіпофізу, розвиток якої супроводжувався збільшенням концентрації соматотропіну в крові. Який процес буде спостерігатися в організмі дитини під впливом надлишку соматотропіну в крові?

@ Посилення синтезу білка, гальмування його розпаду.

Стимуляція процесів глюконеогенезу.

Дистрофічні порушення в епіфізарних хрящах.

Руйнування кісткової тканини, зупинка росту.

Посилення реабсорбції води і Na+ в канальцях нефрону.

#




У жінки, віком 45 років, хворої на тиреотиксикоз спостерігалась субфебрильна температура тіла. Який механізм спричинив це явище?

@ Роз’єднання окисного фосфорування в мітохондріях.

Вплив на центри терморегуляції первинних пірогенів.

Посилення синтезу білків в печінці.

Посилення накопичення глікогену в печінці.

Прямий вплив гормонів щитовидної залози на центри терморегуляції.

#




У дитини, віком 10 років, після перенесеної краснухи утворилися антитела до збудника цієї хвороби. Який вид реактиіності спостерігається в цьому випадку?

@ Специфічний.

Неспецифічний.

Гіперергічний.

Гіпоергічний.

Пасивний.

#




У жінки, віком 60 років, виник напад гострого панкреатиту. Яка ланка в патогенезі цієї хвороби буде головною?

@ Активація протеолітичних ферментів.

Підвищення серкреції інсуліну.

Зниження рівня глікогену в печінці.

Порушення перетравлення білків.

Виділення жирів з калом.

#




У хворого, 38 років, з невралгією трійчастого нерва виник нейрогенний набряк лиця. Який фактор патогенезу наюряку зумовив його виникнення?

@ Підвищення проникливості стінок судин.

Підвищення гідростатичного тиску в капілярах.

Зменьшення онкотичного тиску крові.

Порушення лімфовідтоку.

Позитивний водний баланс.

#




У хворого на гострий вірусний гепатит появились ознаки печінкової коми. Головною ланкою у патогенезі печінкової коми є збільшення вмісту в крові нейротоксичних речовин, насамперед:

@ Аміаку.

Фенолу.

Індолу.

Ацетоїну.

Бутиленгликолю.

#




У хворого 38 років на алкогольний гепатоз виявлено порушення білкового обміну. Які зміни білків крові найбільш вірогідні при цьому?

@ Гіпопротеїнемія.

Гіперпротеїнемія.

Дисгаммаглобулінемія.

Парапротеїнемія.

Дисімуноглобулінемія.

#




У хворого на тиреотиксикоз при обстеженні був встановлений його аутоімунний характер. Ці імунні розлади обусловлює тривалодіючий стимулятор щитовидної залози. До якого класу імуноглобулінів він відноситься?

@ Ig G.

Ig A.

Ig E.

Ig D.

Ig M.

#




У хворого з хронічною правошлунковою недостатністю серця виникли ознаки портальної гіпертензії: розширення вен черевної стінки, асцит, спленомегалія. Який вид портальної гіпертензії має місце у хворого?

@ Надпечінкова.

Внутрішньопечінкова.

Підпечінкова.

Змішана.

-

#




Хворий 40 років страждає хронічним обструктивним бронхитом. Який типовий патологічний процес є головним в клініці цієї хвороби?

@ Запалення.

Гіпоксія.

Алергія.

Патологічна детермінанта.

Стрес.

#




В экспериментальной онкологии применяются различные виды трансплантации опухолей. Какой из указанных видов трансплантации является наиболее эффективным?

@ Аутотрансплантация

Гетеротрансплантация

Изотрансплантация

Гомотрансплантация

Аллотрансплантация

#




Наблюдающаяся у больного лихорадка сопровождается тахикардией, снижением секреторной и моторной функции желудочно-кишечного тракта, активацией фагоцитоза и антителообразования. При этом отмечается усиление гликолиза, отрицательный азотистый баланс, повышение содержания кетонових тел в крови. Как изменяется соотношение процессов образования и отдачи тепла в описанном периоде лихорадки?

@ Теплопродукция равна теплоотдаче

Теплопродукция превышает теплоотдачу

Теплоотдача превышает теплопродукцию

Теплопродукция в пределах нормы, теплоотдача снижена

Теплопродукция ниже теплоотдачи

#




Развившийся в процессе эксперимента анафилактический шок у морской свинки обусловил существенное изменение характера внешнего дыхания. Какой тип дыхания наблюдается в данном случае?

@ Экспираторная одышка

Инспираторная одышка

Периодическое дыхание

Агональное дыхание

Брадипноэ

#




В эксперименте белой крысе парентерально введен 0,1\% раствор адреналина в дозе 1 мл/100г массы тела. Через 60 минут при явлениях острого отека легких животное погибло. Какой механизм развития отека в данном случае является ведущим?

@ Гидродинамический

Онкотический

Осмотический

Лимфогенный

Мембраногенный

#




У больной наблюдаются приступы артериальной гипертензии, сопровождающиеся тахикардией, обильным потоотделением. Для какого из перечисленных опухолевых заболеваний наиболее характерны данные симптомы?

@ Феохромоцитомы

Базофильной аденомы гипофиза

Аденомы клубочковой зоны надпочечников

Аденомы щитовидной железы

Опухоли яичников

#




В результате пожара в ожоговое отделение поступил пострадавший в шоковом состоянии с обширными ожогами туловища 3-4 степени тяжести. Какой из перечисленных факторов является ведущим в развитии ожогового шока?

@ Боль

Плазмопотеря

Инфицирование

Интоксикация продуктами распада поврежденной ткани

Нарушение реологических свойств крови

#




У мужчині 62 лет, работающего в Заполярье руководителем крупного промышленного предприятия, при осмотре стоматологом были обнаружены признаки парадонтоза. Что играет ведущую роль в механизме возникновения данной патологии?

@ Нервно – дистрофический фактор

Возраст больного

Температурный фактор

Недостаток витаминов

Понижение общей физической нагрузки

#




После хронического тонзилита у больного появились признаки развивающейся сердечной недостаточности (тахикардия, одышка, цианоз, отеки нижних конечностей). Каков основной механизм возникшего состояния?

@ Повреждение миокарда

Нарушение коронарного кровотока

Перегрузка сердца объемом

Перегрузка сердца сопротивлением

Нарушение проводящей системы

#




В результате аварии на шахте горнорабочий длительное время находился под завалом, вследствие чего у него развился краш-синдром (синдром длительного раздавливания). Какой из перечисленных факторов является главным в развитии возникающей при этом острой почечной недостаточности?

@ Возникшая токсемия

Боль

Нарушение реологических свойств крови

Образование патологических форм Нв

Уменьшение объема циркулирующей крови

#




При короткостроковій акліматизації до низьких температур навколишнього середовища у тварин відзначається збільшення утворення тепла в організмі. З якими процесами зв'язане збільшення теплопродукції?

@ Скорочувального і не скорочувального термогенезу

Білкового і вуглеводного термогенезу

Печінкового анаеробного термогенезу

Зміни активності гладких м'язів

Жирового і білкового термогенезу.

#




Больной обратился к врачу с жалобами на “голодные” боли в эпигастрии, чувство дискомфорта после приема грубой, жирной, острой пищи. В анамнезе – длительное применение глюкокортикоидов. При обследовании установлен дефект слизистой желудка. Какой механизм является ведущим в возникновении данной патлогии?

@ Недостаток простагландинов

Избыток простагландинов

Избыток гистамина

Повышение тонуса блуждающего нерва

Недостаток гистамина

#




Массивное длительное раздавливание мягких тканей пострадавшего в результате шахтного взрыва привело к появлению у больного олигурии, гипоизостенурии, протеинурии, миоглобинурии, гиперкалиемии, гипонатриемии. Какова причина возникшей при этом почечной недостаточности?

@ Развитие токсемии

Болевое раздражение

Возбуждение симпатической нервной системы

Выброс катехоламинов

Потеря белка

#




Нескольких работников производства по изготовлению взрывчатки доставили в больницу с симптомами отравления. Спектральный анализ крови показал, что это связано с действием бертолетовой соли. Какой тип гипоксии развивается при этом?

@ Гемический

Гипоксический

Дыхательный

Циркуляторный

Тканевой

#




У больного 38 лет в моче обнаружена высокая концентрация белка (16 г/л), цилиндрурия (гиалиновые, зернистые и эпителиальные), гипоизостенурия, моча цвета мясных помоев. Был диагностирован острый диффузный гломерулонефрит. Какова природа данного заболевания ?

@ Аутоаллергическая

Воспалительная

Опухолевая

Гипоксическая

Обменная

#




Хворий з хронічним гломерулонефритом скаржиться на сухість, відчуття гіркоти у ротовій порожнині, запах сечовини. При огляді стоматологом виявлено: ціаноз, набряклість слизової оболонки, на дні порожнини рота та в ретромолярній ділянці вогнища некрозу.В слині збільшена кількість сечовини та залишкового азоту. При якому зменшенні маси діючих нефронів (МДН) найбільш імовірно виявляються такі зміни в ротовій порожнині?

@ Нижче 10\%

Нижче 20\%

Нижче 30\%

Нижче 40\%

Нижче 50\%

#




У пацієнта зі скаргами на кровоточивість ясен при чищенні зубів та жуванні твердої їжі визначили рН ротової рідини – 7,96. При дослідженні кривої зміни рН змішаної слини Стефана при цукровому навантаженні відмічається повільне і виражене зниження рН після цукрового навантаження, а відновлення відбувається за 25 хв (нормальний час). Як оцінити кислотно-лужний стан в ротвій порожнині пацієнта?

@ Посилене кислотоутворення

Понижена нейтралізуюча здатність ротової рідини

Залужнення ротової рідини при цукровому навантажені

Кислотно-лужний стан відповідає нормі

Понижене кислотоутворення

#




Хворий у віці 72 років помер вдома . Черговим лікарем встановлена причина смерті:церебро–васкулярна недостатність. Дайте відповідь на запитання-тканинна гіпоксія у похилому віці є результатом:

@ Всіх наведених факторів

Зменння активності ферментів біологічного окислення

Обезводнення тканин

Нестачею субстратів окислення

#




У хворого К. тривалий час спостерігається виділення гною, кровотечі з ясен. Після обстеження встановлено діагноз акаталезія. Які з механізмів пошкодження клітини зумовлюють ураження слизової ясен та викликають описані прояви?

@ Ліпідні

Кальцієві

Електротно осматичні

Ацидотичні

Нуклеїнові

#




При обстеженні новонародженого встановлено: мікроцефалія, деформація вушних раковин, розщеплення верхньої губи та піднебіння, недорозвиненість обох щелеп, полідактилія. Результати цитогенетичного обстеження:трисомія за 13 парою хромосом. Який найбільш ймовірний діагноз?

@ Синдром Патау

Синдром Дауна

Синдром Шерешевського-Тернера

Синдром Клайнфельтеро

Синдром тиросомії Х

#




У хворого з абцесом щелечно лицевої ділянки після проведеного хірургічного лінування стан покращився. Після операційна рана загоюється первинним натягом. За рахунок яких органел клітини в першу чергу будуть відбуватись процеси регенерації?

@ Рибосом

Мітохондрій

Лізосом

Ендоплазматичної сітки

Комплексу Гольджі

#




Хворий М. 42 р. скаржиться на кровоточивість ясен, загальну слабкість, швидку втомлюваність, часті простудні захворювання. Описані скарги виникають протягом останніх років в кінці зимового періоду недостатність якого вітаміну найімовірнішезумовлює описані прояви?

@ Вітаміну С

Вітаміну РР

Вітаміну В12

Вітаміну Е

Вітаміну D

#




Хворий Ж, 32 р. скаржиться на відчуття загальної слабкості, жару, біль голови, і суглобах, кашель. 5 днів тому мав контакт з хворим на грип t0 - 38.50C. Для якого періоду хвороби характерна описана картина?

@ Періоду розпаду

Латептного

Продронального

Періоду видужання

Рецидиву

#




Пацієнт Х протягом декількох годин знаходився взимку поза приміщенням при температурі зовнішнього середовища - 150С. Яуий компенсаторний механізм був задіяний для підтримкитемператури тіла на сталому рівні у стадії компенсації гіпотермії?

@ М’зове тремтіння

Зниження частоти серцевих скорочень

Поглиблення дихання

Розширення судин шкіри

Зниження інтенсивного обміну речовин

#




Внаслідок дії мутагенного фактора відбулось пошкодження фрагменту ДНК у ядрі клітини. Який процес забезпечено відновлення структури Пошкодженогофрагмента ДНК?

@ Репарація

Трансляція

Транскрипція

Редуплікація

Слонгація

#




У хворого Ч, 12 років, після екстреляції зуба почалась масивна кровотеча з лунки зуба, що не зупинялась звичайними звичайними кровопинними засобами. Лікар припустив, що хворий страждає на гемофілію. Який тип успадкування цієї хвороби?

@ Рецесивний, зчеплений зі статтю

Полігенний

Аутосомно домінантний

Аутосомно рецесивний

Неповне домінування

#




У хворого з гіпертиреозом порушений енергетичний обмін. До яких змін призводить роз’єднання окисного фосфорилювання?

@ Збільшення теплоутворення

Зменшення частки вільного окиснення

Зменшення теплоутворення

Підвищення функціональної активності органів

Збільшення утворення АТФ

#




Який механізм має основне значення в патогенезі гіпоглікемічної коми?

@ Вуглеводне голодування головного мозку

Посилення глікогенолізу

Пригнічення глікогенолізу

Пригнічення гліконеогенезу

Посилення кетогенезу

#




У хворого із серцевою недостатністю розвинулись набряки. Де переважно накопичується рідина у випадку набряку?

@ Міжклітинний простір

Внутрішньоклітинний простір

Позаклітинний простір

Очеревенна порожнина

Внутрішньосудинний прості

#




У молодої жінки після перенесеної гострої вірусної інфекції тривалий час спостерігаються больові відчуття в правій половині обличчя, які посилюються навіть при незначному подразненні? Як називається такий біль?

@ Невралгія

Каузалгія

Фантомний

Рефлекторний

Проекційний біль

#




У жінки 52 років на калькульозний холецистит розвинулася механічна жовтяниця з явищами стеатореї. Вкажіть причину даного явища:

@ Дефіцит жовчних кислот

Дефіцит панкреатичної ліпази

Ферментативна недостатність ентероцитів

Будь-яка з наведених

Жодна з наведених

#




Стоматолог при огляді ротової порожнини хворого відзначив ознаки характерні для гіповітамінозу С. Який патогенетичний механізм пошкодження клітин буде ведучим у даному випадку?

@ Недостатнє утворення антиоксидантів

Внутріклітинним ацидозом

Внутріклітинним алкалозом

Порушенням утворення і-РНК

Активацією фосфоліпази А2

#




Хвора скаржиться на порушення сну, загальну слабкість, роздратованість, тахікардію. Пальпується дифузно-збільшена щитовидна залоза. Яка патологія у хворої?

@ Дифузний токсичний зоб

Тиреоїдит Дібеля

Підгострий тиреоїдит Кервена

Ендемічний зоб

Тиреоїдит Хашімото

#




У туриста, що піднявся на висоту 5200 м, розвинувся газовий алкалоз. Що є причиною розвитку алкалозу?

@ Гіпервентиляція легень

Гіповентиляція легень

Підвищення температури оточуючого середовича

Підвищення атмосферного тиску

Зниження температури оточуючого середовища

#




У ліквідаторів аварії на Чорнобильській АЕС виник геморагічний синдром. Що є основним у патогенезі даного синдрому?

@ Тромбоцитопенія

Порушення структури стінки судин

Підвищення активності факторів системи протизсідання крові

Підвищення фібролітичної активності

Зменшення активності факторів зсідання крові

#




В барокамеру помістили мишу і відтворили модель гірської хвороби, першою ознакою якої була поява корчів. Це пов’язано з тим, що найбільш чутливими до гіпоксії є:

@ Нейрони головного мозку

М’язева тканина

Органи кровотворення

Легені

Нирки

#




В’язень оголосив голодування: не приймав їжу, тільки пив воду. Назвіть характерні зміни в обміні речовин у перший період голодування:

@ Посилення катаболізму білків в м’язовій тканині та глюконеогенезу в печінці

Активація ліполізу в жировій тканині

Посилення розпаду білків життєво важливих органів

Стимуляція утворення кетонових тіл в печінці та розвиток ацидозу

Зменшення катаболізму білків в м’язовій тканині та глюконеогенезу в печінці

#




Внаслідок корабельної аварії матроси опинились на ненаселеному острові, почали голодувати. Які живильні речовини витрачаються при цьому в першу чергу?

@ Вуглеводи

Жири

Нуклеїнові кислоти

Білки

Холестерин

#




У пілота літака на висоті 13000 м раптово розгерметизувалась кабіна. Який вид емболії виник у пілота?

@ Газова

Повітряна

Жирова

Тканинна

Тромбоемболія

#




Лікар при обстеженні хлопчика 3 років виявив підвищення температури до 37,6єС, плями Бєльського-Філатова-Копліка. Який період хвороби спостерігається у хлопчика?

@ Продромальний

Латентний

Інкубаційний

Розпалу хвороби

Кінець хвороби

#




У хворого внаслідок перенесеного плевриту утворились спайки між вісцеральною і парієтальною плеврою. Як називається даний вид патології?

@ Патологічний стан

Патологічний процес

Типовий патологічний процес

Патологічна реакція

Хвороба

#




У хворого лікар виявив ламкість і крихкість кровоносних капілярів, болісність, набряклість ясен, кровоточивість, крововиливи в слизову оболонку, гінгівіт. Дефіцит якого вітаміну спостерігається у хворого?

@ С

В1

В12

Д

Е

#




Хворий після інсульту: свідомість відсутня, шкірні та сухожильні рефлекси пригнічені, зіниці на світло не реагують, тонус скелетних м’язів знижений, температура тіла 35,8єС, дихання Куссмауля, тахікардія. Який вид екстремального стану розвинувся у хворого?

@ Кома

Шок

Колапс

Асфіксія

Запаморочення

#




У хворого на гіпертонічну хворобу з’явився сильний головний біль, шум у вухах, АТ підвищився до 240/160 мм рт. ст. При обстеженні виявлена асиметрія обличчя справа, відсутність довільних рухів, підвищення сухожильних рефлексів і тонусу м’язів правої руки і ноги. Як називається таке порушення рухової функції:

@ Геміплегія

Параплегія

Тетраплегія

Моноплегія

Парапарез

#




Внаслідок травми чоловік втратив 2 л крові і в шоковому стані був доставлений в хірургічний стаціонар. У нього розвинулась анурія. Назвіть головний фактор, який спричинив анурію:

@ Зниження артеріального тиску

Зниження венозного тиску

Втрата білків при кровотечі

Зниження внутрішньониркового тиску

Анемія

#




У жінки 45 років виявлено зниження основного обміну на 40 \%. Дефіцит якого гормону найбільш вірогідно спричинив таке зниження?

@ Трийодтироніну

Соматотропіну

Альдостерону

Тиреокальцитоніну

Інсуліну

#




У хворого лікар виявив в крові підвищення рівня кальцію та зниження – фосфатів. З яким ендокринним порушенням це може бути пов’язаним?

@ Гіперпаратиреоз

Гіпопаратиреоз

Гіпертиреоз

Гіпотиреоз

Гіперальдостеронізм

#




В патогенезі утворення ниркових каменів важливу роль відіграють процеси, які знижують розчинність солей. Назвіть фактор, який не знижує розчинність солей:

@ Підвищення в сечі іонів магнію, цитратів

Зниження в сечі іонів цинку, марганцю

Зниження в сечі іонів магнію, цитратів

Поява в сечі мукопротеїдів

Поява в сечі продуктів розщеплення колагену

#




При ультразвуковому обстеженні нирок у хворого в мисці лівоі нирки виявлено камінь, нирка значно збільшена. Спостерігається порушення сечопуску – анурія. Яке ускладнення нефролітіазу розвинулось у хворого?

@ Гідронефроз

Гломерулонефрит

Пієлонефрит

Некроз миски

Атрофія паренхіми нирки

#




У чоловіка 48 років, який протягом 10 років страждає на виразкову хворобу шлунка, з’явилася часта блювота неперетравленою їжею, періодичні судоми. Рентгенологічно виявлено стеноз пілоруса. Яке ускладнення розвинулося у хворого ?

@ Хлоргідропенічна тетанія

Малігнізація виразки

Перфорація виразки

Флегмона шлунка

Пенетрація виразки в підшлункову залозу

#




Під час проведення езофагогастроскопії у хворого на цироз печінки виникла кровотеча із вен стравоходу. Який патологічний процес не був врахований ендоскопістом ?

@ Позапечінкове портокавальне шунтування

Печінково-ниркова недостатність

Наявність асциту

Внутрішньопечінкове портокавальне шунтування

Капіляризація синусоїдів

#




У молодої жінки через місяць після пологів почала різко падати вага тіла, з’явилася загальна слабкість, зниження функції статевих залоз. Про яке захворювання слід думати?

@ Хвороба Сіммондса

Хвороба Бабінського-Фреліха

Хвороба Аддісона

Базедова хвороба

Хвороба Кушинга

#




У молодої жінки-педагога із скаргами на поганий сон, дратівливість, загальну слабкість і підвищену пітливість під час обстеження виявлено екзофтальм, дифузно збільшену щитоподібну залозу. Яке захворювання має місце?

@ Тіреотоксичний зоб

Спорадичний зоб

Ендемічний зоб

Гострий тіреоїдит

Аденома щитоподібної залози

#




Смерть новонародженого обумовлена гемолітичною хворобою, що підтверджено ядерною жовтяницею на тлі загальної. Назвати вид гемолітичної анемії.

@ Ізоімунна

Гетероімунна

Аутоімунна

Токсична

Механічна

#




Після отруєння ціанідами настає швидка загибель організму внаслідок

@ Незворотної блокади цитохромоксидази

Сповільнення транспорту кисню

Порушення оксигенації гемоглобіну

Затримки дисоціації оксигемоглобіну

Нагромадження вуглекислоти в організмі

#




У хворого діагностовано подагру. Головною ланкою її патогенезу є нагромадження в крові

@ Сечової кислоти

Молочної кислоти

Піровиноградної кислоти

Глютамінової кислоти

Вугільної кислоти

#




У потерпілого внаслідок пригнічення дихального центру від передозування наркотиками виникло порушення кислотно-основної рівноваги, а саме:

@ Газовий ацидоз

Метаболічний ацидоз

Газовий алкалоз

Метаболічний алкалоз

Змішаний ацидоз

#




Юнак 18 років часто звертався до стоматолога з приводу карієсу зубів. Три місяці тому помітив появу крові при чищенні зубів. Нестача якого компонента їжі викликала кровоточивість ясен?

@ Вітаміну С

Вітаміну А

Метіоніну

Кальцію

Фтору

#




Після перенесеної токсикоінфекції з проносом і вираженим зневодненням у дитини 9 років розвинувся синдром мальабсорбції із зниженням засвоєння амінокислот і глюкози. Основна роль в патогенезі порушень активного транспорту їх належить дефіциту іонів

@ Na

К

Mg

Ca

CI

#




У хворого М. вірусний гепатит ускладнився гострою печінковою недостатністю, яка перейшла в кому. Головною ланкою в патогенезі печінкової коми є збільшення в крові нейротоксичних речовин,передусім

@ Аміаку

Фенолу

Індолу

Скатолу

Ацетоїну

#




У жінки 42 років, яка страждала протягом декількох років жовчокам’яною хворобою, розвинулася підпечінкова жовтяниця. Жовтий колір шкіри і слизових з’явився внаслідок відкладення в них

@ Кон’югованого білірубіну

Некон’югованого білірубіну

Уробіліну

Стеркобілірубіну

Білівердину

#




У хворої 20 років після переохолодження з’явилися біль у поперековій ділянці, болючий та почащений сечопуск, підвищення температури тіла до 39 °C. При лабораторному дослідженні крові виявлено лейкоцитоз і прискорення ШОЕ, при дослідженні сечі – лейкоцитурію, протеїнурію, бактеріурію. Яке захворювання найбільш ймовірне в цьому випадку?

@ Пієлонефрит

Аднексит

Сечокам’яна хвороба

Гломерулонефрит

Радикуліт

#




У хворого з гострою нирковою недостатністю кількість виділеної сечі складає 80 мл. Як називається така зміна діурезу?

@ Анурія

Олігурія

Протеїнурія

Лейкоцитурія

Поліурія

#




У жінки, яка протягом 20 років хворіє на хронічний пієлонефрит, відносна густина сечі становить 1,008-1,014. Як називається така зміна діурезу?

@ Гіпостенурія

Ізостенурія

Ніктурія

Поліурія

Олігурія

#




У хворого внаслідок крововиливу у головний мозок відмічається обмеження рухів правими кінцівками, підвищений їх тонус, посилення рефлексів. Який вид порушення рухової функції у даному випадку?

@ Центральний парез

Центральний параліч

Периферичний парез

Периферичний параліч

Мозочкова атаксія

#




Хворий апатичний, малорухливий, відзначається ожиріння, набряки, гіпотонія, брадикардія, зниження обміну речовин. Яке ендокринне захворювання можна запідозрити у хворого?

@ Гіпотиреоз

Апромегалія

Базедова хвороба

Феохромоцитома

Нецукровий діабет

#




У хворої виявлено ожиріння, підвищення артеріального тиску, збільшення рівня цукру в крові, гірсутизм, на шкірі живота – синьо-багрові лінії розтягу. Вкажіть, при якому ендокринному захворюванні можуть виникати дані симтоми:

@ Синдром Іценка-Кушинга

Хвороба Симондса

Хвороба Кона

Базедова хвороба

Хвороба Аддісона

#




В капрдіологічне відділення доставленний хворий А. 45 років,на ЕКГ негативний зубець Р накладаеться на комплекс QRS,диастоличний інтервал після екстрасистоли подовжений.Назвіть вид екстрасистолії?

@ передсердно-шлункова

синусова

передсердна

шлункова

порушення ритму немає

#




Хворий М.,48 років,скаржиться на біль за грудиною,іррадуючі у ліву руку,вгамовані приймом нітрогліцеріну.Лікар виставив діагноз – ішемічна хвороба серця.Який механізм лежить в основі розвитку ішемії міокарду?

@ склеротичний атеросклероз

реперфузійний синдром

активація процесів ПОЛ у міокарді

некроз міокарду

правільної відповіді немає

#




В реанімаційне відділення доставлений хворий К.,шкіряні покрови цианотичні,свідомість відсутня,зі слів лікаря швидкої допомоги хворого витягнули із автомобіля,в якому працював двигун.Який вид гіпоксії у хворого?

@ гемоглобін токсична

малого серцевого викиду

артеріально-гіпоксемічна

венозного примішування

артеріального сбросу

#




Хвора К.,37 років,звернулася у клініку зі скаргами на головний біль,запаморочення,поганий сон,занеміння кінцівок.Останні 6 років працює на заводі газорозрядних ламп у свинцеврму цеху.При обстеженні в аналізі крові кількість еритроцитів та гемоглобіну знижена,вміст сироваточного заліза підвищений в декілька разів.Назвіть вид анемії?

@ залізорефрактерна

залізодефіцитна

анемія Міньковського-Шоффара

гипопластична

метапластична

#




Хвора Л.,38 рокв,звернулася у клініку зі скаргами на головний біль,запаморочення,часте серцебиття,загальну слабкість,підвищену втомлюванность,роздратування.Скарги виникли після перенесенного стресу.При обстеженні АД – 150/90,пульс – 90 за мин.,шкіра долонів волога,визначаеться білий дермографізм.Встановлений діагноз : неврастенія.Який з вищеперерахованних синдромів супроводжує данне захворювання?

@ синдром вегето-судинної дістонії

синдром Кона

синдром Іценко-Кушинга

синдром Бабинського-Фреліха

климактеричний синдром

#




Хвора С.,35 років,поступила до клініки зі скаргами на болісне мочевиделення,непріемні відчуття у низу живота.При вживанни гострої та соленої іжи болі посилюються.При обстеженні:кількість мочи за сутки – 900мл.,щільність мочи – 1020 г/л,реакція сечі – кисла,белок – відсутній,сахар – відсутній,лейкоцити 1-2 в полі зору,еритроцити – одиночні,циліндри – відсутні,сечова кислота – 4,6 ммоль/сут.,в сечі – урати.Визначте вид патології?

@ сечокислий діатез

Сечокам яна хвороба

Хронічний цистіт

Гострий цистіт

Хронічний пієлонефрит

#




При детальному дослідженні каріотипу у хворого на синдром Дауна виявлені соматичні клітини трьох популяцій – нормальні,клітини – трисоміки та клітини-моносоміки.Який механізм лежить в основі геномних мутацій?

@ мозаіцізм

втрата однієї хромосоми

поліплоїдія

робертсонівська перебудовап

центрічний поділ хромосом

#




На консультацію до невропатолога звернулися батьки хлопчика 8 років,зі скаргами на розумову відсталість сина та періодично виникаючи судоми.При обстеженні виявлені нейрофіброми на віях,коньюктиві,роговиці та по ходу ціліарних нервів.Встановлено діагноз нейрофіброматоз(хвароба Реклінгаузена)Вкажіть тип спадкування цієї хвороби?

@ аутосомно-домінантний

аутосомно-рецесивний

сціплений з полом

неповного домінування

полігенний

#




У хворого К.,37 років,середньодольова пневмонія справа супроводжуеться підвищенням температури до 39.Який з нижчеперерахованного є типовим патологічним процесом?

@ гарячка

пневмонія

хвороба

норма

рецидив

#




Хворий З.,36 років звернувся до стоматолога зі скаргами на короткочасні болі в області 5 зуба на верхній щелепі справа.Болі виникають після прийома солодкої їжі.Встановлений діагноз – каріес.Каріес відноситься до

@ патологічного процесу

патологічного стану

хворобою

нормою

патологічною реакцією

#




Хворий К.,переніс запалення легенів з обох боків,супроводжуеться лихоманкою,кашлем.Через тиждень після закінчення курсу антібіотикотерапії скаргі на кашель,на рентгенограммі легенів з обох боків посиленний легочний малюнок.Проходить лікування в кабінеті фізіотерапії.Данний стан треба считати ?

@ неповним виздоровленням

рецидивом

ремісією

ускладненням

повним виздоровленням

#




У клініку привезли людину з тяжкими проявами емфіземи легенів. При опитуванні, було з’ясовано, що він працював на хіміко-фармацевтичному підприємстві, як і його мати, яка померла декілька років тому від захворювання легенів. Лікар припустив наявність спадкової ферментопатії. Яка з перелічених аномалій може бути у цього чоловіка.

@ Недостатність ферменту (-1-антитрипсину

Целіакія

Недостатність ферменту псевдохолінестерази

Акаталаземія

Недостатність ферменту глюкозо-6-фосфатдегідрогенази

#




Чужорідні сполуки (ксенобіотики), що потрапляють в організм людини в процесі життєдіяльності, знешкоджуються у гепатоцитах. Усі наведені реакції належать до І фази біотрансформації ксенобіотиків, за виключенням

@ Реакції кон’югації

Реакції, що каталізуються цитохром Р-450 монооксигеназною системою

Гідролітичної реакції

Реакції окислення амінів

Алкогольдегідрогеназні реакції

#




У дитини спостерігається підвищена збудливість, гіперреактивність, запах плісені від поту та сечі, екземоподібні висипи; кількість фенілаланіну у крові складає 200 мг/л; сеча дає позитивну реакцію Феллінга (з хлоридом заліза). Яке порушення обміну амінокислот у дитини?

@ Фенілкетонурія

Транзиторна тирозинемія

Алкаптонурія

Спадкова тирозинемія

Гомоцистинурія

#




В клиническом анализе крови обнаружен нейтрофильный лейкоцитоз с резким сдвигом лейкоцитарной формулы влево. Какой процесс в организме мог вызвать это явление ?

@ Острая воспалительная реакция

Травматический шок

Аутоиммунная реакция

Глистная инвазия

Аллергическая реакция

#




В отделение интенсивной терапии поступил пациент с симптомами анафилактического шока. Что из нижеперечисленного будет наблюдаться в данном случае ?

@ Гипотензия и тахикардия

Повышение общего периферического сопротивления

Снижение уровня ренина в плазме

Повышение скорости выделения мочи

Снижение гематокрита

#




У девочки-подростка 17 лет в процессе подготовки к вступительным экзаменам в ВУЗ возникла аменорея. Какой этиологический фактор является наиболее важным в развитии данного патологического состояния ?

@ Эмоциональный стресс

Повышение артериального давления

Нерациональное питание

Снижение уровня физической активности

Употребление успокоительных препаратов

#




Пацієнт Ш. 42 років звернувся до стоматолога з приводу рецидивуючого афтозного стоматиту. У патогенезі афтозного стоматиту мають значення алергічні імунопатологічні процеси. Який тип реакцій гіперчутливості з перерахованих може мати найбільше значення при рецидивуючому афтозному стоматиті?

@ Змішана гіперчутливість II, III, IV типів

I типу

II типу

III типу

IV типу

#




У дитини 10 місяців в периферичній крові відсутні зрілі В-лімфоцити. При біопсійном вивченні лімфатичного вузла встановлена відсутність вторинних лімфоїдних фолікулів і плазматичних клітин. Клітинний імунітет не порушений. Така клініко-морфологічна картина характерна для:

@ Вродженої агамаглобулінемії

Атаксії-телеангіоектазії

Комбінованого імунодефіциту

Гіпоплазії тимуса

Т-лімфоцитопенії

#




У хворого 36 років після введення антибіотика з'явився висип, почервоніння і набряклість шкіри. Який вид гіперчутливості характерний для кропив'янки?

@ Гіперчутливість I типу

Реакція типу феномена Артюса

Гіперчутливість II типу

Гіперчутливість III типу

Гіперчутливість IVтипу

#




У хлопчика 3-х років діагностовані множинні фурункули в області спини, на руках. Із слів матері з 8-місячного віку у дитини часто виникали гнійнички на шкірі. Клінічне обстеження показало зниження в сироватці імуноглобулінів всіх класів, відсутність зрілих В-лімфоцитов в периферичній крові. В біоптаті лімфатичного вузла знайдено відсутність реактивних фолікулів і плазматичних клітин. Яке захворювання у дитини?

@ Синдром Брутона

Атаксія Луї-Бару

Синдром Незелофа

Синдром Веста

Комбінований імунодефіцит

#




Жінці 45 років виконана трансплантація нирки, але функція трансплантата прогресивно погіршувалася. Через 1,5 місяці нирку видалили. При гістологічному дослідженні трансплантата виявлено васкуліт, фіброз, лімфоцитарна інфільтрація строми, некроз епітелію канальців. Такі зміни є результатом:

@ Реакции гострого відторгнення трансплантата

Атрофії нирки

Реакції хронічного відторгнення трансплантата

Гострої ниркової недостатності

Реакції найгострішого відторгнення трансплантата

#




У 35-річного чоловіка при гастроскопії знайдено дві „цілуючі” язви пілоричного відділу шлунку, їх краї щільні, підведені над поверхнею слизистої оболонки шлунку, складки слизистої оболонки конвенгують у напрямі розташування дефектів. Шлунковий сік гіперацидний, в крові – гіпергастринемія. В хвості підшлункової залози виявлена пухлина діаметром 2 см. Про яке захворювання можна думати?

@ Синдром Золінгера-Елісона

Ліпоматоз підшлункової залози

Карциноїдний синдром

Виразкова хвороба шлунку

Синдром Вернера-Морісона

#




Мужчина 40 років почав проходити курс повного лікувального голодування. В перший період неекономних витрат в організмі проходить перебудова ендокринної регуляції. Що із перерахованого не характерно для цієї перебудови?

@ Збільшення секреції інсуліну

Збільшення секреції тиреотропного гормону

Збільшення секреції тироксину

Збільшення секреції кортизолу

Збільшення секреції глюкагону

#




У хворого діагностовано порушення кровообігу у системі v. porte в зв’язку з гострою печінковою недостатністю. Ознаками даного синдрому є:

@ Усе нижчеперераховане

Асцит (накопичення рідини у черевній порожнині)

Збільшення печінки

Збільшення селезінки

Розширення поверхневих вен живота

#




У хворого 40 років після видалення лівого наднирника раптово знизився артеріальний тиск до 70/40 мм рт. ст., пульс частий слабкого наповнення, холодний піт, гіпотонія м’язів, судоми. Що найбільш вірогідно призвело до даного стану?

@ Атрофія правого наднирника

Гіпофункція аденогіпофіза

Гіпертрофія правого наднирника

Гіперфункція нейрогіпофіза

Гіпофункція нейрогіпофіза

#




У хворого діагностована феохромоцитома. Вміст якого гормону буде високим ?

@ Адреналіну

Кортизолу

Альдостерону

Реніну

Адренокортикотропну

#




У дитини 13 років зріст 170 см. З надлишком якого гормону це пов’язано?

@ Соматотропного

Адренокортикотропного

Тиреотропного

Оксітоцину

Антидіуретичного

#




Порушення роботи якого відділу ниркового нефрону, який забезпечує процеси фільтрації, спостерігаєтся у хворого на хронічний гломерулонефрит?

@ Клубочки

Проксимальні звивисті канальні

Дистальні звивисті канальні

Юкстагломерулярний апарат

Петля Генує

#




Синтез якого з перерахованих біологічно активних речовин порушено у хворого на хронічний гломерулонефрит, яке регулює еритропоез?

@ Еритропоетин

Ренін

Простагландини

Ангіотензин І

Ангіотензин ІІ

#




У хворого на пієлонефрит порушено реабсорбція органічних іонів у нирках. В якому відділі нефрону це відбувається ?

@ Проксимальних звивистих канальцях

Збиральних трубочках

Дистальних звивистих канальціях

Юкстагломерулярному апараті

Петлі Генує

#




У хворого на цироз печінки спостерігаються набряки нижніх кінцівок та черевної порожнини, патогенетичний механізм набряків у цьому випадку:

@ Онкотичний

Осмотичний

Гіпоксичний

Лімфатичний

Мембраногенний

#




У пацієнта диагностована Базедова хвороба. Надлишок якого гормону спровокувала розвиток даної хвороби?

@ Тироксину

Кальцитоніну

Тиреотропного

Адренокортикотропного

Паратгормону

#




Чоловік 37 років, 4 роки хворіє на хронічний мієлолейкоз. Останнім часом його стан погіршився, в крові з’явилися мієлобласти і почали зникати дозріваючі гранулоцити. Що є причиною змін перебігу хвороби в даному випадку?

@ Прогресія пухлини

Пухлинна трансформація

Промоція пухлинного росту

Метаплазія

Безмежний ріст

#




У хворого на токсикоінфекцію після блювання розвинулося порушення кислотно – основного стану [КОС]. Який вид порушення КОС спостерігається в даному випадку?

@ Видільний алкалоз

Екзогенний алкалоз

Метаболічний ацидоз

Видільний ацидоз

Екзогенний ацидоз

#




У піддослідного кроля спостерігається гіпосекреція шлункового соку. Яким чином це було модульовано?

@ Введенням соматостатину

Введенням гастрину

Введенням гістаміну

Пригніченням продукції простагландинів

Подразненням блукаючого нерва

#




Після пломбування каналу зуба у пацієнта з'явився біль, у розвитку якого, серед іншого, має значення набряк м'яких тканин. Що є головною ланкою патогенезу набряку при гострому запаленні?

@ Ексудація

Еміграція

Альтерація первинна

Альтерація вторинна

Проліферація

#




При гострому пульпіті в місці запалення підвищується температура тканини, виражене її почервоніння. Який розлад місцевого кровообігу є причиною описаних змін?

@ Артеріальна гіперемія

Венозна гіперемія

Тромбоз

Емболія

Ішемія

#




Введення місцевого анестетика викликало, крім аналгезії ділянки в ротовій порожнині, її збліднення. Який розлад місцевого кровообігу з'явився причиною збліднення?

@ Ішемія

Емболія

Тромбоз

Артеріальна гіперемія

Венозна гіперемія

#




Після того, як хлопчик промочив ноги, у нього підвищилася температура тіла, з'явився кашель. Діагностовано гостре респіраторне захворювання. Який фактор є причиною хвороби в даному разі?

@ Інфекція

Охолодження

Гіповітаміноз

Зниження імунітету

Алергія

#




У пацієнта з гіпертрофією міокарда відсутні будь-які ознаки недостатності серця. Маса і розміри серця лишаються незмінними протягом останнього року. Яка стадія розвитку гіпертрофії міокарда найбільш вірогідно має місце у хворого?

@ Завершеної гіпертрофії та стійкої гіперфункції

Аварійної гіперфункції

Поступового виснаження

Прогресуючого кардіосклерозу
#




При обстеженні хворого на гостру ниркову недостатність виявлено гіперкальціемія. Яка ендокринна патологія може бути причиною зазначених порушень?

@ Гіперфункція прищитоподібних залоз

Гіпофункція прищитовидних залоз

Гіперфункція щитоподібної залози

Гіпофункція щитоподібної залози

Гіперфункція наднирників

#




Пацієнту з розповсюдженим кандидозом ротової порожнини рекомендовано подальше обстеження для вирішення питання про можливу ВІЛ- інфекцію. Зі зменшенням кількості яких клітин в крові може бути зв'язаний розвиток кандідозу при СНІДі?

@ CD 4+ Т – лімфоцити

CD 8+ Т- лімфоцити

В -лімфоцити

Макрофаги

Нейтрофіли

#




При фиксации в полости рта металлических пломб и коронок в слюне определяется достоверное увеличение количества ионов металлов: хрома, никеля, марганца. Один из механизмов их токсического действия связан с:

@ Снижением проницаемости клеточных мембран для ионов Са2+

Нарушением фильтрации

Повреждением клеточной мембраны

Повышением активности ионных токов

Повышением проницаемости для ионов С1

#




При заболеваниях почек может возникать протеинурия – патологическое, усиленное выделение (потеря) белков из организма с мочой. При этом возникают отеки тканей. Почему?

@ уменьшается онкотическое давление плазмы крови

Увеличивается онкотическое давление плазмы крови

Уменьшается онкотическое давление тканей организма

Увеличивается онкотическое давление тканей организма

Нарушается регуляция синтеза белков

#




В результате травмы у мальчика 2-х лет возник гемартроз (кровоизлияние в сустав). Родители отмечают, что у мальчика долго кровоточит ссадина, полученная при игре. Из анамнеза известно, что у дедушки по материнской линии были похожие клинические проявления. Какое заболевание и нехватку какого фактора свертывания крови вы заподозрите?

@ Гемофилия А, VIII

Гемофилия А, IX

Гемофилия В, VIII

Гемофилия В, X

Тромбоцитопения

#




Какие последствия для организма может иметь возобновление кровотока в сосудах обширной и длительно ишемизированной области?

@ Активация калликреин-кининовой системы

Ослабление реакции ограниченного протеолиза

Повышение артериального давления

Печеночная недостаточность

Понижение проницаемости сосудистой стенки

#




Почему патология тромбоцитов приводит к кровоточивости?

@ Отсутствует активация, агрегация тромбоцитов и реакция высвобождения БАВ

Повышается в крови концентрация протромбина

Снижено содержание гепарина в крови

Увеличивается образование тромбопластина в крови

Происходит ретракция фибринового сгустка

#




Проявления гипоксического повреждения клетки:

@ Ионный дисбаланс

Алкалоз

Уменьшение активности гидролитических ферментов

Увеличение антиоксидантного потенциала крови

Активация энергозависимых процессов

#




Защитно-приспособительное значение воспаления:

@ Препятствует генерализации процесса

Снижается функциональная активность тканей

Тормозит реакции специфического иммунитета

Снижает лейкопоэз

Тормозит фагоцитоз

#




К проявлениям иммунодефицита по Т-лимфоцитам относятся:

@ Лимфопения

Увеличение активности трансплантационного иммунитета

Отсутствие в крови иммуноглобулинов

Отсутствие в крови компонентов комплемента

Высокая устойчивость к онкологическим заболеваниям

#




В патогенезе аллергических реакций анафилактического и атопического типов имеет место:

@ Активация тканевых базофилов при образовании на их мембранах иммунных комплексов специфических ИГ-Е и аллергена, накопление медиаторов базофилов

Недостаточное накопление антител, преимущественно представленных ИГ-Е

Избыточное накопление антител, преимущественно представленных ИГ-М

Активация системы комплемента, фагоцитов и тромбоцитов

Длительная циркуляция в крови иммунных комплексов

#




Какие факторы способствуют быстрому заживлению слизистой полости рта:

@ Антиэрозивный фактор

Паротин

Амилаза слюны

Высокое содержание кальция в слюне

Низкая васкуляризация слизистой полости рта

#




Всасывание токсических продуктов в ротовой полости приводит к:

@ Сенсибилизации организма

Анемии

Гипогликемии

Гиперазотемии

Гипоазотемии

#




Причины гипосаливации:

@ Обезвоживание организма

Отравление парасимпатомиметиками

Повреждение слизистой полости рта

Глистные инвазии

Гипергидратация организма

#




Какие заболевания сопровождаются патологическими проявлениями в ротовой полости?

@ В12-дефицитная анемия

Фенилкетонурия

Мочекислый диатез (подагра)

Гипоксическая гипоксия

Гипокапния

#

Крок 1. Стоматологія. 2005 - 2006 рік

6.0 Патологічна фізіологія

1

Відомо, що розвиток ракових захворювань часто супроводжується стійким субфебрилітетом Що в цьому випадку відіграє роль первинного пірогену?

@Власні ушкоджені клітини

Бактерії

Віруси

Інтерлейкін 1

Інтерлейкін 2

#

2

В школі під час санації учнів 7 класу у одного хлопчика виявлена відсутність бокових різців. Як з’ясувалося, таким же дефектом страждає і батько цього хлопчика. Така патологія є:

@Молекулярно-генетичною з домінантним типом успадкування

Молекулярно-генетичною зчепленою з Х-хромосомою

Молекулярно-генетичною зчепленою з У-хромосомою

Хромосомною аберацією

Хворобою зі спадковою схильністю

#

3

В науковій лабораторії досліджують культуру клітин, клонованих з молігнізованої клітини. Ведучим патогенетичним механізмомушкоджених клітин в цьому випадку є:

@Нуклеїновий

Ліпідний

Протеїновий

Кальцієвий

Ацидотичний

#

4

При стоматологічному огляді у пацієнта 37 років, було констатовано відсутність 1 лівого верхнього премоляру. За словами пацієнта, зуб було видалено два роки тому внаслідок парадонтозу. У хворого є:

@Патологічний стан

Патологічна реакція

Патологічний процес

Хронічний процес

Ускладнення

#

5

Хворий, 65 років, хворіє ішемічною хворобою серця протягом п’яти років. Яка найбільш імовірна причина ішемії у хворого?

@Атеросклероз

Емболія

Вади серця

Спазм вінцевих артерій

Здавлювання артерій судиною

#

6

У дитини після прогулянки в лісі в шкіру проник кліщ і розвинулося запалення:

@Проліферативне

Гнійне

Геморагічне

Фібринозне

Альтеративне

#

7

У хворої з тиреотоксикозом був поставлений діагноз: хронічний гломерулонефрит. Який основний симптом свідчить про ушкодження клубочкової мембрани у даної хворої?

@Протеїнурія

Гіперглікемія

Лейкоцитурія

Циліндурія

Аміноацидурія

#

8

У виявлено пухлину слинної залози. Яка принципова відмінність пухлинних клітин?

@Неконтрольованість поділу

Швидкість поділу

Зниження функціональних можливостей

Збільшення функціональних можливостей

Зміна напрямку диференціювання

#

9

У хворого на ентерит, що супроводжувався значною діареєю, спостерігається зменшення кількості води в позаклітинному просторі, збільшення її в середині клітин та зниження осмолярності крові. Як називають таке порушення водно-електролітного обміну?

@Гіпоосмолярна гіпогідратація

Гіперосмолярна гіпогідратація

Осмолярна гіпогідратація

Гіпоосмолярна гіпергідратація

Гіперосмолярна гіпергідратація

#

10

Хворому з метою обезболення, ввели розчин новокаїну. Через декілька хвилин у хворого розвинулась задишка, тахікардія, втрата свідомості. Який шок розвинувся у хворого?

@Анафілактичний

Кардіогенний

Геморагічний

Травматичний

Опіковий

#

11

У експериментальної тварини після тривалого голодування виміряли дихальний коефіцієнт – 0,7. Який період голодування?

@Другий

Перший

Третій

Четвертий

П’ятий

#

12

В патогенезі багатьох стоматологічних захворювань, зокрема карієсу, важливу роль відіграють процеси, що виникають на поверхні емалі. Слина справляє на емаль зубів захисний вплив. Котрий з перелічених впливів не відноситься до захисних?

@Демінералізуючий

Очищувальний

Буферний

Антибактеріальний

Ремінералізуючий

#

13

Хворий П., 43 років був скерований в нейрохірургію із основним діагнозом – еозинофільна аденома гіпофізу. Чим клінічно проявляється це захворювання?

@Акромегалією

Гіпофізарним гігантизмом

Інфантилізмом

Гіпофізарним нанізмом

Карликовістю

#

14

У хворого Б., 50 років виникла масивна тромбоемболія легеневої артерії, внаслідок чого розвинувся шок. Яка патогенетична форма шоку виникла?

@Кардіогенний

Первинний гіповолемічний

Травматичний

Септичний

Анафілактичний

#

15

Експериментальній тварині введено розчин калію ціаніду. Який тип гіпоксії буде спостерігатися у тварини?

@Тканинна

Гіпоксична

Циркуляторна

Гемічна

Дихальна

#

16

У хворого з переломом стегнової кістки в ділянці хірургічної шийки з’явились ознаки гострої правошлуночкової недостатності по причині емболії легеневої артерії. Якого виду емболія зумовила такі симптоми?

@Жирова

Тромбоемболія

Газова

Повітряна

Тканинна

#

17

У хворої з хронічним тромбофлебітом глибоких вен нижніх кінцівок після фізичного навантаження з‘явились ознаки емболії легеневої артерії. Який розлад периферичного кровообігу спричинив цей процес?

@Емболія мігруючим тромбом

Венозна гіперемія

Емболія повітрям

Артеріальна гіперемія

Емболія тканинним детритом

#

18

У хворого з облітеруючим атеросклерозом проведено десимпатизацію стегнової артерії в ділянці стегнового трикутника. Який вид артеріальної гіперемії виник внаслідок операції?

@Нейропаралітична

Реактивна

Метаболічна

Нейротонічна

Робоча

#

19

У хворого, що поступив в ургентному порядку в приймальний покій чергової лікарні з підозрою на гострий апендицит, скарги на болі в правій клубовій ділянці, нудоту, відсутність апетиту за останній місяць, закрепи, які змінюються проносами, субфебрильну температуру. В розгорнутому аналізі крові: Нв-100г./л., Лейкоцити-9,5х109/л. В формулі крові - еозінофілія. ШОЄ-30мм./год. Загальний аналіз сечі в нормі. Про запалення якого характеру можна думати?

@Алергічне

Гнійне

Хронічне специфічне

Серозне

Геморрагічне

#

20

З анамнезу пацієнта, в якого виявлено рак сечового міхура, відомо, що останні 10 років він працював в цеху, що спеціалізувався на виготовленні барвників для виробництва яких застосовувались аміноазосполуки і аміни. Яка саме речовина могла сприяти цьому захворюванню?

@(-нафтиламін

Ортоаміноазотолуол

Диметиламіноазобензол

Діетилнітрозамін

Метилнітрозосечовина

#

21

Видалення зуба у пацієнта з хронічним персистуючим гепатитом ускладнилось тривалою кровотечею. Яка причина геморрагічного синдрому?

@Порушення утворення тромбіну

Порушення утворення тромбопластину

Порушення утворення фібрину

Збільшення синтезу фібриногену

Посилення фібрінолізу

#

22

Під час операції з приводу гранульоми в ділянці правого верхнього різця виникла кровотеча, яку вдалось зупинити тільки через 3 години. В аномнезі хворого хронічний лімфолейкоз. Що було ймовірно причиною кровотечі?

@Тромбоцитопенія

Тромбоцитопатія

Лімфоцитоз

Лейкопенія

Еозінофілія

#

23

У хлопчика, який штучно вигодовувався коров’ячим молоком, виявлено блідість шкірних покровів, глосит, гінгівіт, та появу карієсу. При обстеженні крові: еритроцити-3,3х1012/л., гемоглобін-70г./л., кольоровий показник-0,5. У мазку крові: гіпохромія, мікроцитоз, пойкілоцитоз. Яка анемія спостерігається у хворого?

@ Залізодефіцитна

В12-фолієводефіцитна

Серпоподібно-клітинна

Гемолітична

Таласемія

#

24

Чоловік 70 років, хворіє на атеросклероз судин нижніх кінцівок та ішемічну хворобу серця. Під час обстеження виявлено порушення ліпідного складу крові. Надлишок яких ліпопротеїнів є головною ланкою в патогенезі атеросклерозу?

@Низької щільності

Дуже низької щільності

Високої щільності

Проміжної щільності

Хіломікронів

#

25

У пацієнта з стенозом правого атріовентрикулярного отвору спостерігається запалення печінки, варікозне розширення вен черевної стінки, стравоходу, асцит, набряки нижніх кінцівок. Що викликало такі клінічні ознаки?

@Хронічна недостатність серця за правошлуночковим типом

Гостра серцева недостатність

Хронічна недостатність серця за лівошлуночковим типом

Кардіопатія

Тотальна недостатність серця

#

26

Хлопчик народився доношеним. Огляд дитини виявив аплязію вилочкової залози, дефект верхньої губи і відкриту артеріальну протоку. При обстеженні спостерігається відсутність Т-лімфоцитів. Який синдром розвинувся в дитини?

@Синдром Ді Джорджі

Синдром Брутона

Синдром Луї-Барра

Синдром Чедіака-Хігасі

Імунодефіцит швейцарського типу

#

27

Після лікування виразкової хвороби шлунка при фіброгастроскопії на місці виразки виявлено рубець. До якої патофізіологічної категорії можна віднести дане явище?

@Патологічний стан

Патологічний процес

Патологічний рефлекс

Захисна реакція

Патологічна реакція

#

28

Хворий потрапив до лікарні після іонізуючого опромінення зі скаргами на блювання, анорексію, біль в різних ділянках живота, наявність крові у калі, підвищення температури тіла, в’ялість. Для якої форми гострої променевої хвороби характерна клінічна картина?

@Кишкової

Кістковомозкової

Церебральної

Змішаної

Токсемічної

#

29

При зовнішньому огляді грудної клітки пацієнта спостерігається тупий епігастральний кут, широкі міжреберні проміжки, горизонтальне розміщення ребер (бочкоподібна грудна клітка). Якому типу конструкції за Саго відповідає конституція пацієнта?

@Дихальному

М’язовому

Травному

-

Мозковому

#

30

Хворий А. 40р. звернувся до лікаря-стоматолога з приводу різкого болю в ділянці нижньої щелепи. При Rtg- обстеженні встановлено перелом нижньої щелепи. В анамнезі часті патологічні переломи кісток. Що лежить в основі такого захворювання?

@Надмірна продукція паратгормону

Збільшена продукція СТГ

Зменшене виділення паратгормону

Зменшене виділення АКТГ

Збільшена продукція тироксину

#

31

До стоматолога-ортопеда звернувся чоловік 32 років, якому півроку тому була проведена екстракція зуба. Кольор слизової оболонки на місці екстракції не відрізняється від кольору других ділянок ротової порожнини. Яке це явище?

@Патологічний стан.

Патологічна реакція.

Патологічний процес.

Типовий патологічнийй процес.

Компенсаторна реакція.

#

33

Чоловіка, віком 60 років, у якого встановлено хронічний гепатит, спостерігалась почтійна кровотеча з носа і ясен, і спонтанноз’являлись геморагічні висипання на шкірі і слизових оболонках. Наслідком чого є ці явища?

@Зменьшенням синтезу протромбіну і фібріногену.

Підвищенням вмісту амінотрансфераз плазми крові.

Зменьшенням утворення сироваткових альбумінів.

Появою в крові макроглобулінів і криоглобулінів.

Зменьшенням в крові рівня холінестерази.

#

34

У жінки, віком 40 років, у якої було виявлено дифузний токсичний зоб, спостерігалось постійне підвищення температури тіла. Який механізм зумовив це явище?

@Роз’єднання окисного фосфорування в мітохондріях клітин.

Посилення розпаду глікогену в клітинах печінки.

Посилення катаболізму білка в клітинах.

Підвищення збудливості клітин нервової системи.

Підвищення чутливості клітин до катехоламінів.

#

35

У жінки, віком 45 років, було виявлено ендемічний зоб. Який механізм викликав гіперплазію щитовидной залози у цієї хворої?

@Посилення вироблення тиротропіну.

Посилення вироблення тироксину.

Посилення всмоктування йоду.

Посилення гідратації шкіри і підшкірної клітковини.

Посилення вироблення катехоламінів.

#

36

Жінка 28 років, що хворіє на цукровий діабет, народила дитину зі збільшеною масою тіла, ураженням кістково-м’язової, серцево-судинної і центральної нервової системи. У віці 2 роки у дитини був виявлений цукровий діабет внаслідок порушення внутрішньоутробного розвитку. Який вид порушення внутрішньоутробного розвитку у дитини найбільш вірогідний?

@Специфічна фетопатія.

Неспецифічна фетопатія.

Бластопаті.

Ембріопатія.

Гаметопатія.

#

37

В основі багатьох захворювань лежать типові патологічні процеси, такі як запалення, пухлина, голодування. Які основні закономірності властиві типовим патологічним процесам?

@Поліетіологічність і монопатогенетичність.

Поліетіологічність.

Монопатогенетичність.

Моноетіологічність.

Поліпатогенетичність
38

Жінка 45 років хворіє на рак лівої молочної залози. На лівій руці є ознаки недостатності лімфатичної системи – набряк кінцівки, збільшення лімфовузлів. Яка форма недостатності лімфообміну спостерігається у хворої?

@Механічна недостатність.

Динамічна недостатність.

Резорбційна недостатність.

Змішана недостатність.

-

#

39

Одним з маханізмів неспецифічного імунологічного захисту організму людини є фагоцитоз мікроорганізмів лейкоцитами. Які з перелічених речовин відіграють найбільш важливу роль при знищенні мікробів в фагосомах нейтрофілів?

@Гідролази лізосом.

Кисневі радикали.

Лізоцим.

Лактоферин.

Катіонні білки.

#

40

Однією з гематологічних ознак алергічних хвороб людини являється еозинофілія, яка захищає організм від дії біологічно активних речовин (БАР) – медіаторів алергії. Який механізм захисної дії еозинофілів являється найбільш важливим?

@Руйнування БАР.

Припинення секреції БАР.

Пригнічення БАР.

Захист клітки-мішені від впливу БАР.

Дуактивація і запобігання впливу БАР.

#

41

Хвора 65 років страждає жовчокам’яною хворобою. Останній час з’явились ознаки ахолічного синдрому внаслідок обтурації жовчних шляхів, з явищами харчового голодування. Засвоєння яких компонентів їжі буде порушено найбільше?

@Жирів.

Білків.

Вуглеводів.

Вітамінів.

Електролітів.

#

42

У хворого 49 років виявлено: набряк лиця, значна протеінурія, гіпопротеінемія, диспротеінемія, гіперліпідемія.

@Нефротичний синдром.

Січекам’яна хвороба.

Простатит.

Пієлонефрит.

Цистіт.

#

43

В експерименті на тварині дослідник змоделював асептичне запалення введенням подразнювального розчину в черевну порожнину. Як у данному випадку можна розглядати поняття хвороби?

@Як типовий патологічний процес.

Як хворобу в данного індівідума.

Як нозологічну одиницю.

Як філософстке узагальнення.

Як патологічний стан.

#

44

Дівчинка 5 років була доставлена у інфекційну лікарню каретою швидкої допомоги з діагнозом грип. При обстеженні: t тіла 39,90С, сухий надривний кашель, задишка, гіперемійована, гаряча на дотик шкіра, ЧСС - 130 в хв., ЧДД - 28 в хв. Який за ступенем підвищення температури тіла вид гарячки розвинувся у дитини?

@Висока.

Субфебрильна.

Помірна.

Гіперпіретична.

Низька.

#

45

В експерименті тварині провели оперативне втручання – перев’язування загальної жовчної протоки, що супроводжувалось холестазом та виникненням жовтяниці. Який вид жовтяниці за механізмом виникнення розвинувся у експериментальної тварини?

@Холестатична.

Гемолітична.

Печінкова.

Транспортна.

Кон’югаційна.

#

46

Під час сінокосу у селянина з’явилась кропив’янка, підвищилась температура тіла, що супроводжувалось лейкоцитозом. Який вид лейкоцитозу спричиняється в даному випадку?

@Еозинофілія

Базофілія

Нейтрофільоз

Лімфоцитоз

Моноцитоз

#

47

У хворого виявили базофільну аденому гіпофіза. Це приводить до підвищеної продукції таких гормонів:

@Глюкокортикоїдів

Вазопресину

Інсуліну

Адреналіну

Соматотропіну

#

48

У хворого поліурія до 8 л на добу, полідипсія. Рівень глюкози в крові 4,5ммоль/л. Лабораторні дослідження можуть дати в даному випадку такий результат:

@Пригнічення утворення вазопресину

Підвищення секреції вазопресину

Зниження вмісту в крові адреналіну

Підвищення секреції окситоцину

Зниження рівня інсуліну в крові

#

49

У пацієнта з синдромом Іценка-Кушинга виявлена гіперглікемія, глюкозурія. Назвіть основний механізм гіперглікемії у даного хворого.

@Стимуляція глюконеогенезу

Стимуляція глікогенолізу в печінці

Стимуляція глікогенолізу в м’язах

Стимуляція всмоктування глюкози в кишечнику

Стимуляція синтезу глікогену

#

50

У пацієнта маляра розвинувся контактний дерматит. Основна особливість цієї патології в тому, що з антигеном взаємодіють Т- лімфоцити, які виділяють лімфокіни. До якої групи відноситься фактор бласттрансформації?

@Група Б

Група А

Група В

Група Д

Група Г

#

51

Пацієнта ужалила бджола, після чого в місці пошкодження виник біль і набряк. Назвіть головний фактор запального набряку в даному випадку:

@Підвищення проникності судин

Зниження онкотичного тиску крові

Підвищення онкотичного тиску в осередку запалення

Підвищення осмотичного тиску в осередку запалення

Підвищення гідростатичного тиску в артеріолах

#

52

Відомо, що в патогенезі гарячки важливе значення має підвищення установочної точки центру терморегуляції. Це здійснюється завдяки дії на центр:

@Простагландинів Е1 та Е2

Простагландинів Д та Н

Інтерлейкіну - 1

Екзопірогенів

Циклооксигенази

#

53

Згідно з пропозицією ВООЗ первинний цукровий діабет розділяють на інсулінозалежний і інсулінонезалежний. Етіологічним фактором інсулінозалежного цукрового діабету є:

@Пошкодження В – клітин вірусами

Відсутність рецепторів до інсуліну

Міцний зв’язок інсуліну з білком

Пошкодження гіпофіза

Висока активність інсулінази

#

54

Відомо, що при цукровому діабеті у хворих частіше зустрічаються запальні процеси, знижена регенерація та уповільнюється загоювання ран. Причиною цього являється:

@Зниження протеосинтезу

Підвищення ліполізу

Прискорення гліконеогенезу

Зниження ліполізу

Посилення катаболізму

#

55

При алергічних реакціях негайного типу виникає дегрануляція тканинних базофілів, які виділяють біологічно активні речовини. Однією з таких є:

@Гістамін

Ацетилхолін

Система комплементу

Профібринолізин

Фактор Хагемана

#

56

При цукровому діабеті I типу з тривалим перебігом у хворого розвинулась мікроангіопатія. З активацією якого метаболічного шляху пов’язане пошкодження непосмугованих м’язових клітин судинної стінки при цукровому діабеті?

@Сорбітолового

Пентозного

Аргінінового

Міоінозитолового

Циклу Кребса

#

57

У хворого діагностовано аліментарну дистрофію, яка супроводжується набряками. Назвіть основний фактор розвитку набряку при цьому.

@Гіпопротеїнемія

Затримка Na+ в організмі

Підвищення проникності кровоносних судин

Порушення утворення лімфи в тканинах

Підвищення гідростатичного тиску в капілярах.

#

58

У слабо підготовленого альпініста в горах розвинулась гіпоксія:

@Гіпоксична

Респіраторна

Циркуляторна

Тканинна

Гемічна

#

59

У хворого розвинувся гіповітаміноз В12, який привів до тканинної гіпоксії. Для цієї гіпоксії характерно:

@Зменшення артеріо - венозної різниці по кисню

Збільшення артеріо – венозної різниці по кисню

Зменшення напруги кисню в артеріальній крові

Збільшення кисневої ємності крові

Зменшення кисневої ємності крові

#

60

На завершальному етапі марафонської дистанції спортсмен знепритомнів, впав. Лікарі діагностували коматозний стан. Яка кома найбільш вірогідно розвинулась при цьому?

@Гіпоглікемічна

Гіперосмолярна

Ацидотична

Гіперглікемічна

Токсична

#

61

У хворого з запаленням відмічається припухлість шкіри, болісність, почервоніння ділянки запалення, підвищення місцевої температури. Який розлад периферичного кровообігу має місце в даному випадку?

@Артеріальна гіперемія

Венозна гіперемія

Стаз

Тромбоз

Ішемія

#

62

При обстеженні хворого з жовчно - кам’яною хворобою визначили застій жовчі в печінці і в протоках, жовчні камені в жовчному міхурі. Вкажіть основний компонент жовчних каменів, які утворюються в такому стані:

@Холестерин

Білірубін

СаСО3

Тригліцериди

Жовчні кислоти

#

63

При аналізі ЕКГ виявлено випадіння деяких серцевих циклів PQRCT. Наявні зубці і комплекси не змінені. Назвіть вид аритмії.

@Синоатріальна блокада

Миготлива аритмія

Атріовентрикулярна блокада

Передсердна екстрасистола

Внутрішньопередсердна блокада

#

64

У хворого на ессенціальну артеріальну гіпертензію розвинувся гіпертонічний криз, що привело до нападу серцевої астми. Який механізм серцевої недостатності є провідним в даному випадку?

@Перевантаження серця підвищеним опором

Перевантаження серця збільшеним об’ємом крові

Абсолютна коронарна недостатність

Пошкодження міокарда

Порушення надходження крові до серця

#

65

Чоловік, хворий на дальтонізм, вступив до шлюбу із здоровою жінкою, батько якої хворів на дальтонізм, мати здорова, серед її родичів хворих на дальтонізм немає. Визначте вірогідність народження в цій родині хворих на дальтонізм дітей.

@50\%

25\%

0\%

75\%

100\%

#

66

В силосной яме обнаружен труп мужчины. Судебно-медицинское исследование показало высокое содержание в крови сульфгемоглобина. Какая форма гипоксии имела место?

@Гемическая

Гипоксическая

Респираторная

Циркуляторная

Тканевая

#

67

У человека, заболевшего крупозной пневмонией, в течении нескольких часов температура тела поднялась до 39,50С. За счет чего, прежде всего, произошло повышение температуры тела?

@Снижения теплоотдачи и увеличения теплопродукции

Увеличения теплопродукции

Разобщения окисления и окислительного фосфорилирования

Увеличения несократительного термогенеза

Увеличения сократительного термогенеза

#

68

Ребенок 5-ти лет болеет коревой краснухой. При обследовании крови обнаружено увеличение общего количества лейкоцитов до 13·109/л. Лейкоцитарная формула выглядит следующим образом: Б-0, Э-1, Мц-0, Юн-0, Пя-2, Ся-41, Л-28, М-28. Как называется описанное явление?

@Моноцитоз

Агранулоцитоз

Лимфоцитоз

Эозинопения

Нейтропения

#

69

Человека, занимающегося работой под водой на глубине 100 метров, в результате аварийной ситуации пришлось поднимать на поверхность в срочном порядке. При этом у него отмечались боли в суставах, зуд кожи, нарушение зрения, нарушение нервно-мышечной координации. Как называется возникшее у работника состояние?

@Кессоная болезнь

Газовая эмболия

Гипероксия

Десатурация

Взрывная декомпрессия

#

71

Женщина 43 лет госпитализирована с жалобами на боли в правом подреберьи, кожный зуд. При длительном обследовании установлено: снижение болевой чувствительности и скорости свертывания крови, желтушность кожных покровов, брадикардия, гипотония. Что является наиболее вероятной причиной данного симптомокомплекса?

@Ретенционная желтуха

Повышение гемолиза эритроцитов

Сахарный диабет

Паренхиматозная желтуха

Печеночная желтуха

#

72

Больной 32-х лет в начальной стадии гипертонической болезни жалуется на частые мочеиспускания. Что лежит в основе этого явления при данном заболевании?

@Повышение тонуса отводящих артериол почек

Повышение тонуса приводящих артериол почек

Понижение тонуса отводящих артериол почек

Снижение осмолярного давления крови

Повышение онкотического давления крови

#

73

У крыс, холодящихся в состоянии стресса, повышен мышечный тонус, артериальное давление, повышено содержание глюкозы в крови, усилена секреция кортикотропина и кортикостероидов. В какой фазе стресса находятся эти животные?

@Фазе противошока

Стадии истощения

Фазе шока

Эректильной стадии

Терминальной стадии

#

74

Реализация общего адаптационного синдрома осуществляется преимущественно через нейроэндокринную систему. Какому из звеньев этой системы принадлежит ведущая роль в патогенезе развивающейся реакции?

@Гипофизарно-адреналовому

Гипофизарно-тиреоидному

Гипофизарно-адреногенитальному

Гипофизарно-инсулярному

Гипофизарно-юкстагломерулярному

#

75

У 10-месячного ребенка при осмотре обнаружено: кожа и волосы обесцвечены, тонус мышц повышен. Обследование ребенка показало наличие в крови повышенной концентрации фенилаланина. Данное заболевание может быть:

@Наследственным по рецесивному типу

Наследственным по доминантному типу

Наследственным, сцепленным с полом

Врожденным

Приобретенным

#

76

У больного Л. после приема недоброкачественной пищи развился многократный понос. На следующий день у него снизилось артериальное давление, появилась тахикардия, экстрасистолия. Ph крови составляет 7,18. Эти нарушения являются следствием:

@Негазового ацидоза

Газового ацидоза

Негазового алколоза

Газового алколоза

Метаболического алколоза

#

77

У больного Р., страдающего хронической почечной недостаточностью, обнаружено в крови повышение уровня остаточного азота до 40 мг\%, больше половины которого составляет мочевина. Выявленная гиперазотемия является:

@ Ретенционной

Печеночной

Продукционной

Резидуальной

Смешанной

#

78

У больного с желтухой в анализе крови обнаружено увеличение общего билирубина за счет непрямой его фракции. Моча и кал интенсивно окрашены. Каков наиболее вероятный механизм указанных нарушений?

@ Повышенный гемолиз эритроцитов

Затруднение оттока желчи из печени

Повреждение паренхимы печени

Нарушение образования прямого билирубина

Нарушение превращения уробилиногена в печени

#

79

Больному было проведено исследование мочи по Зимницкому, которое выявило, что относительная плотность её в течении суток колебалась в пределах 1,006 – 1,012. О нарушении какого процесса в почках свидетельствуют полученные данные?

@Канальцевой реабсорбции

Клубочковой фильтрации

Канальцевой секреции

Канальцевой экскреции

-

#

80

У хворого з крупозною пневмонією з’явилося часте поверхневе дихання. Воно має назву

@Тахіпное

Гіперпное

Брадипное

Диспное

Апное


#

81

Густина сечі пацієнта з хронічною нирковою недостатністю складає 1,009 і практично не змінюється після прийому їжі та рідини. Цей симптом позначають як

@Ізостенурія

Гіпостенурія

Полакіурія

Олігурія

Ніктурія

#

82

При лабораторному обстеженні дитини виявлено, що вміст білка у сечі складає 60 г/л. Наявні білки з великою молекулярною масою. Цю протеїнурію можна вважати

@Органічною

Аліментарною

Дегідратаційною

Холодовою

Позанирковою

#

83

Мама звернулась до лікаря з приводу запалення слизової оболонки рота в її дворічної дитини, яке з’явилось півроку тому, і при цьому звичайні протизапальні засоби не покращували стан. Виявилось, що дитину рано почали згодовувати козячим молоком. Вірогідною причиною порушень є недостатність вітаміну

@В12

С

В2

Е

Р

#

84

Після травми передньої ділянки шиї у постраждалого з’явились фібрилярні скорочення м’язів обличчя під час легкого постукування по шкірі спереду від зовнішнього слухового отвору.Найбільш вірогідною причиною може бути зниження в крові

@Са

К

Na

Mg

P

#

85

У хворого 64 років виявлено рак слизової оболонки порожнини рота. При огляд – значне руйнування кутних зубів, незадовільний гігієнічний стан порожнини рота. Курить на протязі 50 років, вживає алкоголь, в анамнезі – робота на ЧАЕС. Найвірогідніше причиною розвитку раку було:

@Іонізуюче випромінювання

Подразнення слизової оболонки їжею

Вживання алкоголю

Куріння

Незадовільний гігієнічний стан порожнини рота

#

86

У піцієнтки 23 років виявлено, що після використання нової губної помади з’явилися набряк і свербіння губ, а через 2 дні – кірочки на червоній облямівці губ. Який тип алергіїчної реакції найбільш вірогідний?

@Анафілактична

Цитотоксична

Імунокомплексна

Сповільнена

Стимулююча

#

87

Після опромінення у хворого з’явився яскраво виражений геморагічний синдром на слизовій оболонці порожнини рота у вигляді дрібних крововиливів і петехіїй. Виникла гематурія. Механізм появи геморагічного синдрому полягає в

@Зменшенні кількості тромбоцитів

Активації фібринолізу

Збільшенні вмісту гепарину

Пошкодженні судин

Зменшенні вмісту фібриногену

#

88

Найбільш вірогідним проявом СНІДу у порожнині рота будуть:

@Саркома Капоші

Карієс

Пульпіт

Пародонтит

Хейліт

#

89

У хворого діагностовано грип. Після прийому антипіретиків стан його різко погіршився: свідомість потьмарена, АТ 80/50 мм рт.ст., пульс 140 уд/хв, температура тіла 36,8 °C. Яке ускладнення виникло у даного хворого?

@Колапс

Гіпертермія

Гіповолемія

Ацидоз

Алкалоз

#

90

В ендокринологічному відділенні обласної лікарні знаходиться хворий із скаргами на швидку втомлюваність, поганий апетит. З анамнезу відомо: тривалий час лікувався кортикостероїдами з приводу бронхіальної астми. Об’єктивно: АТ 90/60 мм рт.ст., гіперпігментація шкіри. Яке захворювання можна запідозрити у хворого? @Хвороба Аддісона

Хвороба Іценка-Кушінга

Хвороба Кона

Хвороба Сімондса

Хвороба Грейвса

#

91

До лікаря звернувся хворий із скаргами на спрагу, схуднення, частий сечопуск. При обстеженні виявлено гіперглікемію, глюкозурію. Рівень якого гормону знижений у хворого?

@Інсуліну

Глюкагону

Тироксину

Адреналіну

Вазопресину

#

92

У хворого у порожнині рота спостерігається повна атрофія сосочків язика, язик червоний, “лакований”, укритий виразками. В аналізі крові відмічається лейкопенія, зсув формули вправо, К.П. –1,3, мегалоцити, тільця Жолі. Для якої патології це характерно?

@ В12 - дефіцитна анемія.

Залізодефіцитна анемія.

Залізорефрактерна анемія.

Агранулоцитоз.

Гемолітична анемія.

#

93

У хворого має місце гінгівіт, деструкція пародонта, виразки слизової порожнини рота. При зменшенні яких клітин крові це спостерігається?

@ Нейтрофілів.

Лімфоцитів.

Моноцитів.

Еозинофілів.

Базофілів.

#

94

У хворого гнійна рана щелепно-лицьової ділянки. Які з перехованих клітин відіграють головну роль в фазу регенерації раньового процесу?

@ Фібробласти.

Нейтрофіли.

Моноцити.

Еозинофіли.

Лімфоцити.

#

95

У хворого після видалення зуба кровотеча зупинилась через годину після операції. Яке захворювання крові найвірогідніше запідозрити у даного хворого, якщо відомо, що проба Кончаловського – негативна?

@ Гемофілія

Хвороба Верльгофа.

Хвороба Глянцмана

Хвороба Шенляйна - Геноха.

Хвороба Вакеза – Ослера.

#

96

У хворого при госпіталізації діагностована двобічна пневмонія. Яка патогенетична форма порушень зовнішнього дихання має місце у даного хворого?

@ Вентиляційно-рестриктивна.

Дифузійно-пневмонозна.

Первинно-дискінетична.

Обструктивна.

Дифузійно - рестриктивна.

#

97

Хворий скаржиться на часті нудоти, блювоту, відчуття важкості в епігастральній ділянці. Порушення якої функції шлунка лежить в основі цього синдрому?

@ Євакуаторної.

Секреторної.

Моторної.

Інкреторної.

Екскреторної.

#

98

Хворий звернувся зі скаргою на зубний біль. При обстеженні у нього діагностовано пульпіт. Який фактор відіграв головну патогенетичну роль у формуванні больового синдрому в даному випадку?

@ Підвищення внутрішньотканинного тиску в пульпі зуба. .

Пошкодження пульпи .

Неадекватне подразнення гілочки нижньощелепного нерва.

Активація одного із компонентів системи комплементу.

Дія інтерлейкінів.

#

99

У чоловіка віком 50 років, в результаті травми вибито багато зубів на нижній і верхній щелепі. Що виникає в організмі чоловіка внаслідок цього?

@ Патологічний стан.

Латентний період.

Рецидив.

Період ремісії.

Патологічний процес.

#

100

Жінка 55 років 10 років тому перенесла міокардит, є недостатність мітрального клапану. В даний час скарг немає. Гемодинаміка в межах норми. Якому поняттю загальної нозології відповідають дані умови?

@ Патологічний стан.

Патологічний процес.

Типовий патологічний процес

Патологічна реакція.

Компенсаторна реакція.

#

101

При підйомі в гори у рятівника розвинулось психомоторне збудження, запаморочення, тахікардія, яка чергувалася з апное. Яке порушення кислотно-основного стану розвинулося у рятівника?

@ Газовий алкалоз.

Негазовий алкалоз.

Газовий ацидоз.

Негазовий ацидоз.

Метаболічний алкалоз.

#

102

Хворий у віці 60 років звернувся до стоматолога. Лікар виявив карієс 3-х зубів. Куди можна віднести дану патологію?

@ Патологічний процес.

Патологічний стан.

Рецидив.

Латентний період.

Продромальний період.

#

103

Жінка, яка лічила зуби з приводу карієса, із-за ускладнення погодилася на видалення зуба. Що з патологічних явищ у жінки можна віднести до поняття “патологічний стан”?

@ Відсутність зуба.

Підвищення температури.

Почервоніння.

Припухлість

Карієс.

#

104

На прийомі у стоматолога під час екстракції зуба частина твердих тканин потрапила у верхні дихальні шляхи (трахею). Який вид порушення зовнішнього дихання розвинеться у хворого?

@ Обструктивний.

Первинно-дискінетичний.

Вентиляційно-рестриктивний.

Дифузійно-рестриктивний.

Пневмонозно-рестриктивний.

#

105

Під час пломбування зуба у хворої відмічалась рефлекторна гіперсалівація. Яке порушення кислотно-основного стану може розвинутись у пацієнта?

@ Видільний ацидоз.

Видільний алкалоз.

Газовий ацидоз.

Метаболічний ацидоз.

Газовий алкалоз.

#

106

При обстеженні хворого на атрофічний гастрит виявлено: еритроцити — 2,0 • 1012/л, гемоглобін — 5,2 ммоль/л, колірний показник — 1,3, лейкоцити — 4,0 • 109/л, тромбоцити — 180 • 109/л. Дефіцит якої речовини може бути причиною анемії в хворого?

@Вітаміну В12

Фолієвої кислоти

Вітаміну К

Заліза

Цинку

#

107

Під час чергового обстеження хворого на глютенову ентеропатію [целіакію] виявлено: еритроцити — 2,0 • 1012/л, гемоглобін — 6,0 ммоль/л, колірний показник — 1,5, лейкоцити — 4,1 • 109/л, тромбоцити — 200 • 109/л. Як, за патогенезом, можна характеризувати анемію у хворого?

@Дизеритропоетична, фолієводефіцитна

Дизеритропоетична, залізодефіцитна

Гемолітична, ферментопатія

Гемолітична, гемоглобінопатія

Гемолітична, мембранопатія

#

108

У працівників підприємства по виробленню килимових покриттів відмічені риніт, дерматит, бронхіальна астма. За якою класифікацією ці хвороби можуть бути віднесені до одної групи?

@Патогенетична

Топографо – анатомічна

Екологічною

За віком

За статтю

#

109

Усі контактери з хворим на дифтерію були обстежені бактеріологічно. В одному випадку виявлено збудника дифтерії. Жодних скарг на стан здоров’я обстежена людина не має. Який період хвороби має місце?

@Латентний

Продромальний

Розпал

Кінець

Одужання

#

110

В другому періоді повного голодування з водою втрати маси тіла тварини склали 10\%. Які органи і тканини найінтенсивніше втрачають свою масу ?

@Жирова тканина

М’язова тканина

Печінка

Нирки

Нервова тканина

#

111

Введення знеболюючого пацієнту перед екстракцією зуба призвело до розвитку анафілактичного шоку, який супроводжувався розвитком олігурії. Який патогенетичний механізм зумовив зменшення діурезу в даній клінічній ситуації?

@Зниження гідростатичного тиску в капілярах клубочків

Підвищення гідростатичного тиску в капсулі Шумлянського-Боумена

Пошкодження клубочкового фільтру

Збільшення онкотичного тиску крові

Зменшення кількості функціонуючих нефронів

#

112

На рентгенограмі хворого є затемнення в області правої легені. За результатами туберкулінової реакції та бактеріологічного дослідження встановлено діагноз “Туберкульоз легень”. Імунні реакції якого типу є причиною хронічної альтерації і запалення легень в хворого?

@Клітинні

Анафілактичні системні

Анафілактичні локальні

Гуморальні цитотоксичні

Зумовлені імунними комплексами

#

113

Через декілька хвилин після внутрішньовенного введення препарату артеріальний тиск у пацієнта знизився до 70/30 мм. рт. ст. Який з хімічних медіаторів анафілаксії спричинює вазоділатацію і шок?

@Гістамін

Гепарин

Інтерлейкіни

Фактор хемотаксису еозинофілів

Фактор хемотаксису нейтрофілів

#

114

При обстеженні хворого на ревматоїдний артрит, який приймає глюкокортикоїди, виявлена лімфопенія. Як можна охарактеризувати функціональний стан імунної системи пацієнта?

@Імунодефіцит вторинний

Імунодефіцит первинний

Імунодефіцит вроджений

Толерантність до ауто антигенів

Анафілаксія

#

115

Через тиждень після введення гетеросироватки в пацієнта наявні гарячка, набряклі суглоби, висипи. При обстеженні: лімфоцитоз, збільшення ШОЕ, протеїнурія. Яке запалення є наслідком надмірного створення розчинних імунних комплексів в хворого?

@Гостре імунне

Хронічне імунне

Гостре неімунне

Хронічне неімунне

Гіпоергічне

#

Крок 1. Стоматологія. 2006 - 2007 рік

6.0 Патологічна фізіологія

1

В клинику доставлена женщина 65 лет, потерявшая много крови в результате дорожно- транспортного проишествия. На предотвращения какого патологического явления должны быть в первую очередь направлены лечебные мероприятия?

@Гипохолемии

Анемии

Снижения активности свертывающей системы

Тканевой гипоксии

Лейкопении

#

2

У больного Х. при исследовании картины крови виявлено повышенное сожержание базофилов (5\% ) и эозинофилов (12\%) на фоне лейкоза. Для какой разновидности лейкозов это характерно?

@Хронического миелолейкоза

Миелобластного

Хронического лимфолейкоза

Лимфобластного

Мегакариоцитарного

#

3

У мужчині 46-лет при флуорографии был обнаружен туберкулез легких. Дальнейшее клиническое обследование показало специфические изменения состава белой крови,а именно:

@Лимфоцитозов

Моноцитозов

Нейтрофилию

Базофилию

Эозинофилию

#

4

Рабочий завода химических реактивов поступил в стационар с острым отравлением нитробензолом. Анализ крови показал : эритроцитов – 2,8 ?1012/л; Hb-80г/л ; Ц. П.-08, ретикулоцитов -19\%. В мазке крови: анизотитоз, пойкилоцитов, единичные нормоциты. Осмотическая резистентность эритроцитов: максим.-0,38\%; миним.-0,55\%. О какой форме анемии может идти речь?

@Гемолитической

Апластической

Железодефицитной

Гипопластической

Пернициозной

#

5

У 19 летнего юноши, страдающего в течение пяти лет дифузным гломерулонефритом, отмечается повышение уровня артериального давления. Что в данном случае является ведучим патогенетическим фактором повышения АД?

@Избыток ангиотензинаII

Избыток катехолеминов

Избыток антидиуретического гормона

Повышение тонуса симпатической нервной системы

Избыток альдостерона

#

6

У больного вирусным гепатитом на второй неделе заболевания появилось расстройство сна , головная боль, агрессивность , нестерпимый зуд кожи. Объективно отмечается снижение АД, свертываемости крови, рефлекторной деятельности, брадикардия .Чем обусловлены эти изменениия?

@Холалемией

Гиперлипемией

Уробилинемией

Гиперхолестеринемией

Стеркобилинемией

#

7

У работницы химического предприятия , перенесшей отравление, выявлено повышение в плазме крови содержания общего билирубина за счет непрямого. В кале и моче высокое содержание стеркобилина. Уровень прямого билирубина в плазме крови находится в пределах нормы. О каком виде желтухи может идти речь?

@Гемолитической

Обтурационной

Печеночной

Паренхиматозной

Механической

#

8

На ЭКГ больного виявлено сокращение длительности интервала R-R. С какими изменениями работы сердца это связано?

@Увеличением частоты сердечних сокращений

Ростом силы сокращений

Падением частоты сердечных сокращений

Уменьшением силы сокращений

Замедлением частоты и снижением силы сердечных сокращений

#

9

В инфекциооное отделение поступил больной с жалобами на боль в правом подреберье, общую слабость, желтушность кожи, обесцвеченный кал. Объективно: склеры и кожа желтушны, t-39?C, печень увеличена, кал ахоличный. Был поставлен диагноз инфекционного гепатита Какой стадии болезни соответствуют описанные явления?

@Разгару болезни

Инкубационному периоду

Латентному периоду

Продромальному периоду

Исходу болезни

#

10

Пострадавший во время землетрясения находился 7 суток в каменном завале без пищи и воды. Какой тип голодания возник в этом случае?

@Абсолютное

Полное

Количественное

Качественное

Частичное

#

12

На прием к врачу –стоматологу пришла пациентка с признаками истерического поведения. Какой вид одышки может развиться у нее при выполнении врачебных менипуляций в ротовой полости?

@Полипноэ

Брадипноэ

Гиперпноэ

Стенотическое дыхание

Эупноэ

#

13

При исследовании мочи у врача- стоматолога , собранной в конце рабочего дня,обнаружено содержание белка 0,7г/л . В утренней моче таких измениений не обнаружено. Это явление можно классифицировать так:

@Функциональную протеинурию

Органическую протеинурию

Неселективную протеинурию

Внепочечную протеинурию

Ложную протеинурию

#

14

При обследовании у мужчины 24 лет в моче выявлены следующие изменения: относительная плотность мочи-1001, белка, сахара- нет, лейкоциты 1-2 в п/зр, эритроциты- не обнаружены. Соли- оксалаты, небольшое колличество. Суточный диурез – 10 литров.Пациент жалуется на сильную жажду, частое мочеиспускание. Что является наиболее вероятной причиной данного заболевания?

@Гиперсекреция антидиуретического гормона

Гиперсекреция вазопрессина

Гипосекреция вазопрессина

Относительная инсулиновая недостаточность

Гиперсекреция альдестерона

#

16

Після тривалої зими і нестачі вітамінів у людини почали розшатуватись зуби, з'явились порушення діяльності імунітету, відмічені аутоімунні реакції. Ці процеси відбуваються за участю клітин:

@ Т- і В - лімфоцитів.

Еритроцитів.

Нейронів.

Стовбурових клітин.

Тромбоцитів.

#

18

Жінка 52 р., хвора на рак нижньої щелепи, пройшла курс променевої терапії. Розмір пухлини зменшився. Який з наведених механізмів ушкодження клітини найбільш обумовлює ефективність променевої терапії?

@Утворення вільних радикалів

Гіпертермія

Лізис NK-клітинами

Тромбоз судин

Мутагенез

#

19

Чоловік 25 р. був ужалений бджолою. При наданні першої медичної допомоги йому було введено адреналін. Для зупинення розвитку якого з наведених нижче імунологічних механізмів була проведена наведена маніпуляція?

@генералізованої анафілаксії

вивільнення інтерлейкінів з макрофагів

зв’язування з антирецепторними антитілами

місцевого утворення імунних комплексів

активації системи комплементу

#

20

У хлопчика 3 р. після введення туберкуліну на передпліччі на другий день з’явилася ділянка затвердіння діаметром 15 мм (позитивна реакція Манту). Найбільш ймовірно, що це явище є наслідком реакції гіперсенсибілізації (за класифікацією за Джеллом і Кумбсом)

@IV типу

I типу

II типу

III типу

V типу

#

22

Хворий М.,44 роки страждає на бронхіальну астму. У час довготривалий нападів експіраторної задишки, спостерігається розвиток:

@Обструктивної гіповентіляції

Рестриктивної гіповентиляції

Дизрегуляторної гіповентиляції

Рефлексу Китаєва

Порушення дихання не має

#

23

На прийомі у лікаря-стоматолога хворий раптово втратив свідомість, через хвилину у нього розвинулись судоми, які мали характер короткочасних скорочень та розслаблень окремих груп м’язів, що швидко слідують одне за одним. Який вид судомів розвинувся у хворого?

@Клонічні

Тонічні

Тоніко-клонічні

Атетоз

Гемібалізм

#

24

Хворому при переливанні крові визвали гарячку несумісною групою крові. Наведіть повну класифікацію цього виду гарячки?

@Неінфекційна, первинна, екзогенна, природня

Інфекційна,вторинна,екзогенна,штучна

Інфекційна,первинна,екзогенна,природня

Неінфекційна,вторинна,екзогенна,штучна

Неінфекційна,вторинна,ендогенна,штучна

#

25

Хворому Ж.,22 роки, встановили діагноз: паренхіматозна жовтяниця. Вкажіть основні групи причин, які призводять до паренхіматозної жовтяниці?

@Інтоксикація

Дискенезія жовчних протоків

Переливання крові несумісних груп

Обтурація жовчних протоків

Компресія жовчних протоків

#

26

Хвора К., скаржиться на зниження температури тіла, набряки, м'язову слабкість, діарею. З анамнезу відомо, що мала місце довготривала діета. При обстеженні виявлено: гіпопротеінемія, зменьшення основного обміну, атрофічні синдроми. Назвіть вид голодування?

@Білкове

Жирове

Вуглеводне

Мінеральне

Водне

#

27

Хворий К.,30 років, скаржиться на біль в попереку. В аналізі мочі визначається білок,лейкоцити. Діагноз-піелонефрит.Проявом якого процесу є остуда при пієлонефриті?

@Гарячка

Інфекція сечових шляхів

Синдрому ДВЗ

Порушення відтоку сечі

Запалення

#

28

Пацієнт К.,45 років скаржиться на підвищення температури тіла. Температурна крива різко підвищилась в першу добу до 40(С, трималась 7 днів, коливаясь у межах одного градусу. Який тип температурної кривої спостерігався у хворого?

@ebris continua

ebris hectika

ebris recurens

ebris intermentis

Правильної відповіді не має

#

29

В приймальне відділення лікарні доставлений хворий М.,50 років, який страждає на цукровий діабет. Якє з нижчеперерахованних визначень характеризує стан гіперглікемії?
@Підвищення рівня глюкози в крові

Зниження рівня глюкози в крові

Виведення глюкози із сечею

Збільшення вмісту кетонових тіл у крові

Виведення кетонових тіл із сечею

#

30

На прийомі у лікаря-стоматолога, після введення новокаїну у хворого розвинувся анафілактичний шок. В основі розвитку гемодинамічних розладів шоку лежать такі механізми, крім

@Гіперволемія

Зменшення ОЦК

Зменшення хвилинного об’єму серця

Зменшення загального периферійного опору

Порушення реологічних властивостей крові

#

31

У хворого К.,46 років, лікарем-стоматологом діагностовано злоякісна пухлина. Назвіть послідовність етапів процесу метастазування?

@Інтравазація,дисемінація,екстравазація.

Дисемінація,екстравазація,інтравазація

Дисемінація,інтравазація,екстравазація

Екстравазація,дисемінація,інтравазація

Інтравазація,поглинання,дисемінація

#

32

Хворий К.,29 років, лікував зубний біль “народним методом” та помилково випив гемічну отруту, метгемоглобинутворююч.Розвинулася стійка гемічна гіпоксія. Дайте визначення метгемоглобіну?

@Гемоглобін, в якому залізо тривалентне

З’єднання гемоглобіну з сірководнем

З’єднання гемоглобіну з чадним газом

З’єднання гемоглобіну з метанолом

Гемоглобін, в якому залізо двохвалентне

#

33

У чоловіка, якому 2 тижні назад були встановлені коронки зубів, десневі сосочки і слизова оболонка десневого краю гіперемовані, припухлі, кровоточать. Чим є виявлений у чоловіка гінгівіт?

@Нозологічна одиниця.

Типовий патологічний процес.

Патологічний стан.

Патологічна реакція.

Синдром.

#

34

У чоловіка, якому 2 тижні назад були встановлені коронки зубів, десневі сосочки і слизова оболонка десневого краю, гіперемовані, припухлі, кровоточать. Диагностован гінгівіт. Чим є гіперемія при гінгівіті у чоловіка?

@Симптом.

Синдром.

Нозологічна одиниця.

Типовий патологічнийй процес.

Патологічний стан.

#

35

У чоловіка, віком 30 років, у якого виникло запалення легень, спостерігалось збільшення загальної кількості лейкоцитів у крові. Який характер має лейкоцитоз у цьому випадку?

@Лейкоцитоз реактивного характеру.

Лейкоцитоз пухлинного характеру.

Лейкоцитоз відносного характеру.

Лейкоцитоз переросподільного характеру.

Лейкоцитоз спадкового характеру.

#

36

У альпіністів, які тривалий час знаходились у високогірному районі, спостерігалось збільшення кількості еритроцитів – більше 6х1012/л і гемоглобіну – більше 170 г/л у крові. Який механізм спонукав виникненню цього явища?

@Посилення продукції еритропоетину нирками.

Посилення гемолізу еритроцитів у кровоносному руслі.

Посилення здатності тканини утілізувати кисень.

Посилення процесів безкисневого збільшення енергії.

Посилення внутрішньоклітинного гемолізу еритроцитів.

#

37

У чоловіка віком 25 років, після тривалого находження на сонці при високій вологості повітря, підвищилась температура тіла до 390С. Який патологічний процес спостерігається у цьому випадку?

@Гіпертермія.

Інфекційна гарячка.

Гіпотермія.

Неінфекційна гарячка.

Опікова хвороба.

#

38

Чоловік 26 років перебуває в торпідній стадуї шоку внаслідок автомобільної аварії. Кількість лейкоцитів крові 3,2х109/л. Який головний механізм в розвитку лейкопенії?

@Перерозподіл лейкоцитів у судинному руслі.

Пригнічення лейкопоезу.

Порушення виходу зрілих лейкоцитів з кісткового мозку в кров.

Руйнування лейкоцитів у кровотворних органах.

Підвищення виділення лейкоцитів з організму.

#

39

В експерименті у білих щурів в наслідок харчування зерном, зараженому плісневими грибами, тому що перезимувало на полі, виникла алейкія. Які зміни крові найбільш вірогідні при цьому?

@Панцитопенія.

Лейкопенія.

Анемія.

Тромбоцитопенія.

Лимфоцитопенія.

#

40

В лікарню поступив хворий зі скаргами на виникнення болю і поносу, що часто змінюється запором під впливом негативних емоцій. Обстеження виявило у хворого синдром подразненої кишки. З патологією якого гормона травної системи пов’язаний цей синдром?

@Мотилін.

Гастрин.

Секретин.

Гастроінгібуючий пептид.

Вазоактивний інтестинальний пептид.

#

41

У хворого після переливання еритроцитарної маси виник шок і появились ознаки гострої ниркової недостатності. Що являється провідним механізмом розвитку ГНН в даному випадку?

@Порушення клубочкової фільтрації.

Порушення канальцевої реабсорбції.

Порушення канальцевої секреції.

Порушення сечевиділення.

Порушення інкреторної функції нирок.

#

42

У хворої зі справжньою поліцитемією розвинулось порушення мікроциркуляції, обумовлене зміною реологічних властивостей крові, а саме:

@Збільшення в’язкості

Гіпокоагуляцією

Гіперальбумінемією

Гіпофібриногенемією

Зменшення в’язкості

#

43

У хворої 12-ти років виявлено накопичення рідини в черевній порожнині. При пункції отримано мутний пунктат світло жовтого кольору, відносної щільності 1,029 з вмістом білка 0,39 г/л. В осаді велика кількість клітин, переважно нейтрофілів. Який характер рідини, отриманої при пункції?

@Гнійний

Гнилявий

Геморагічний

Фібринозний

Серозний

#

44

Дівчинка 5 років – безколірні волосся, брови, вії; шкіра біла з рожевим відтінком; райдужна оболонка червонява. Розлади метаболізму якого субстрату спричинилися до розвитку описаної патології у дитини?

@Тирозину

Фенілаланіну

Гомогентизинової кислоти

Триптофану

Пуринових основ

#

45

Розвиток І стадії гарячки характеризується наступними змінами терморегуляції:

@ Обмеження тепловіддачі, збільшення теплопродукції

Збільшення тепловіддачі, зменшення теплопродукції

Збільшення тепловіддачі, збільшення теплопродукції

Зменшення теплопродукції, зменшення тепловіддачі

Теплопродукція дорівнює тепловіддачі

#

46

У хворого на стоматит хлопчика 2 р. виникла помірна гарячка. З виділенням яких БАР це може бути пов'язано?

@Інтерлейкіну-1, простагландинів

Лізосомальних ферментів

Лізоциму, інтерферону

Гістаміну, серотоніну

Гепарину, брадикініну

#

47

Працівник пекарні отримав термічний опік руки ІІ ступеня. Які з перерахованих механізмів є специфічними для цього ушкодження клітин?

@ Коагуляція білків та ліпопротеїдних структур

Ацидоз клітини

Порушення цілісності структури клітини

Утворення вільних радикалів

Контрактури клітин

#

48

Піддослідну тварину не годували і давали пити. Тварина збуджена, агресивна. Основний обмін дещо підвищений, дихальний коефіцієнт – 1,0. Який період голодування?

@Неекономного витрачання енергії

Максимального пристосування

Термінальний

Неповного голодування

#

49

Згідно з патогенезом кишкової непрохідності для цієї патології не характерно:

@Збільшення ШОЕ

Блювота

Зневоднення

Гіповолемія

Атонія кишок

#

50

Який синдром не буде характерним для прояву у хворого кістково-мозгової форми гострої променевої хвороби в період удаваного благополуччя?

@Гіпоксичний

Геморагічний

Гематологічний

Кишковий

Астенічний

#

51

Що не є причиною розвитку у хворого порушення легеневої перфузії?

@Зменшення дихальної поверхні легень

Шок

Лівошлуночкова серцева недостатність

Стеноз отвору мітрального клапану

Правошлуночкова серцева недостатність

#

52

У хворого із хронічним гепатитом виникли порушення в системі крові. Що не є типовим?

@Еритроцитоз

Анемія

Коагулопатія

Лейкопенія

Тромбоцитопенія

#

53

Любитель дайвінгу занурився на глибину 20 м, використовуючи для дихання киснево-гелієву суміш, в якій концентрація кисню становила 10\%. З’явились ознаки токсичної дії кисню на організм. Що є причиною цього?

@Утворення вільних радикалів

Утворення БАР запалення

Збільшення проникності судин

Спазм бронхіол

Пошкодження клітинних мембран

#

54

Алергія у людей з відхиленнями в системі імунітету буває при таких обставинах, крім

@Ослаблення утворення біологічно активних речовин

Сповільнене руйнування антигену

Посилена продукція IgЕ

Зрив імунологічної толерантності

Полегшення проникнення алергену в організм

#

55

Гостра недостатність серця супроводжується такими порушеннями, крім:

@Анемія

Зниження хвилинного об’єму серця

Підвищення венозного тиску

Задишка

Гіпоксія міокарда

#

56

Фактори, що стимулюють клітинний поділ, можуть порушувати регуляцію його і сприяти канцерогенезу. До них відносяться

@Естрогени

Гормони надниркових залоз

Інсулін

Ренін

Ацетилхолін

#

57

В розвитку набряку при ураженнях печінки основну роль відіграє:

@Зниження колоїдно-осматичного тиску крові

Підвищення колоїдно-осматичного тиску в тканинах

Підвищення гідростатичного венозного тиску

Підвищення проникності капілярних судин

Порушення відтоку лімфи

#

58

Хворий 23 років звернувся з скаргами на зниження маси тіла, швидку психічну і фізичну втомлюваність, поганий апетит, дисфункцію шлунково-кишкового тракту, прогресуючу гіперпігментацію шкіри, зниження артеріального тиску. При дослідженні було виявлено туберкульоз наднирникових залоз, що призвело до розвитку:

@Хвороби Аддісона

Хвороби Іценко-Кушинга

Хвороби Кона

До підвищення функції мозкової речовини наднирникових залоз

Адрено-генітального синдрому

#

59

У хворої 40 років після тривалого прийому лівоміцитину, спостерігається наступні зміни в крові: знижена кількість еритроцитів і гемоглобіна, відсутні ретикулоцити, знижена кількість нейтрофілів на фоні відносного лімфоцитозу. Тромбоцитопенія і я к наслідок геморагічний синдром. Яка з перерахованих анемій розвинулась у хворої

@Апластична

Залізодефіцитна

В12-дефіцитна

Гемолітична

Білково-дефіцитна

#

60

Хворий поступив до клініки зі скаргами на загальну слабкість, підвищення температури тіла, болі в кістках та суглобах. У крові: Лейкоцити - 68хГ/л Лейкоцитарна формула: мієлобласти - 62\% мієлоцити - 4\% метамієлоцити - 2\% палочкоядерні - 2\% сегментоядерні - 18\% еозинофіли - 1\% базофіли - 2\% лімфоцити - 7\% моноцити - 2\% Для якої з перелічених патологій характерні такі зміни білої крові:

@Мієлобластний лейкоз

Мієлоцітарний лейкоз

Хронічний лімфолейкоз

Лейкемоїдна реакція мієлоїдного типу

Гострий недиференційованний лейкоз

#

61

У жінки 50 років у травмі постійно виникаї набряк Квінке. Який патогенетичний механізм набряку?

@Дефіцит інгібітора компліменту С-1

Дефіцит гаммаглобулинів у крові

Дефіціт по Т-лімфоцитам

Порушення фагоцитозу

СНІД

#

62

Хлопець 14 років, високого зросту з ївнухоподібними пропорціями тіла, вузькі плечі, широкий таз, підшкірна основа розвинута надмірно, оволосіння в ділянці лобка по жіночому типу, статевий член нормальних розмірів, інтелект значно знижений. Яку патологію слід підозрювати?

@Синдром Клайнфельтера

Трисомія по Х хромосомі

Хвороба Дауна

Хвороба Шерешевськьго-Тернера

Синдром Едвардса

#

63

Ліквідатор аварії на ЧАЕС отримав дозу опромінення 15 Гр. Який найбільш імовірний діагноз.

@Кишкова форма гострої променевої хвороби

Кістково-мозкова форма гострої променевої хвороби

Церебральна форма гострої променевої хвороби

Хронічна променева хвороба

#

64

При дослідженні сечі і крові хворої дитини виявляється високий рівень гомогентизинової кислоти. Для якого захворювання це притаманне?

@Алкаптонурія

Тірозиноз

Альбінізм

Фенілкетонурія

Галактоземія

#

65

У хворого, який знаходився в гастроєнтерологічному, відділенні проведено дослідження шлункового соку і встановлено збільшення базальної та максимальної секреції. Який найбільш імовірний діагноз:

@Виразкова хвороба 12-палої кишки

Хронічний атрофічний гастрит

Рак шлунка

Невроз

Перніціозна анемія

#

66

Відомо, що для артеріальної гіперемії притаманне збільшення кровонаповнення органа за рахунок надмірного надходження крові артеріальними судинами. Які з позначених нижче функціональних змін притаманні для неї?

@Підвищення місцевої температури

Підвищення венозного кровотоку

Зменшення артеріального кровообігу

Спазм судин

Ділатація судин

#

Крок 1. Стоматологія 2000-2010 рр

6.0 Патологічна фізіологія

1

У букальних мазках епітелію жінки виявлено в ядрі клітини 2 тільця Барра. Це характерно для синдрому:

@ Трисомія статевих хромосом

Трисомія 21-ї хромосоми

Трисомія 13-ї хромосоми

Трисомія по У-хромосомі

Моносомія статевих хромосом

#

2

При різних запальних процесах у людини в крові збільшується кількість лейкоцитів. Ця закономірність є проявом:

@ Адаптації.

Регенерації.

Репарації

Трансплантації

Дегенерації.

#

3

Під час розтину трупа новонародженого хлопчика виявлена: полідактилія, мікроцефалія, незарощення верхньої губи та верхнього піднебіння, а також гіпертрофія паренхіматозних органів. Сукупність вказаних вад відповідає синдрому Патау. Яке порушення у генетичному апараті є причиною даної патології?

@ Трисомія 13-ої хромосоми

Трисомія 18-ої хромосоми

Трисомія 21-ої хромосоми

Нерозходження статевих хромосом

Часткова моносомія

#

4

У хворого взята кров для аналізу. Її дані показують, що 30% еритроцитів мають неправильну форму. Як називається цей процес?

@ Патологічний пойкілоцитоз

Анізоцитоз

Фізіологічний пойкілоцитоз

Макроцитоз

Мікроцитоз

#

5

Стінки судин мають досить значні морфологічні розбіжності у будові середньої оболонки. Чим зумовлена поява специфічних особливостей будови цієї оболонки у різних судинах?

@ Гемодинамічними умовами.

Впливом органів ендокринної системи.

Регуляцією з боку центральної нервової системи.

Індуктивним впливом нейронів вегетативних гангліїв.

Високим вмістом катехоламінів у крові.

#

6

В пунктате миелоидной ткани ребенка 6 лет обнаруживаются клетки, в которых в процессе дифференцировки происходит пикноз и удаление ядра. Назовите вид гемопоэза, для которого характерны данные морфологические изменения.

@ Эритроцитопоэз.

Тромбоцитопоэз.

Гранулоцитопоэз

Лимфоцитопоэз

Моноцитопоэз

#

7

У мазку крові хворого після перенесеного грипу виявлено 10% округлих клітин розмірами 4,5 7 мкм, які мають велике кулясте ядро, базофільно забарвлену цитоплазму у вигляді вузької облямівки навколо ядра. Який стан крові вони характеризують?

@ Лімфоцитопенію.

Тромбопенію.

Лейкопенію.

Лімфоцитоз.

Моноцитопенію

#

8

У хворого 14 років, спостерігається порушення сутінкового бачення. Якого вітаміну недостатньо в організмі?

@ А

В1

В6

С

В12

#

9

Студентові дано препарати двох мазків. На одному- все поле зору вкрите еритроцитами, на другому визначаються формені елементи крові різного ступеня зрілості. Що це за мазки?

@ Кров і червоний кістковий мозок людини

Кров і лімфа

Кров жаби і кров людини

Кров і мазок жовтого кісткового мозку

Мазок жовтого і червоного кісткового мозку

#

10

У хворого на пневмонію у загальному аналізі крові виявлено зростання загальної кількості лейкоцитів. Як називається це явище?

@ Лейкоцитоз

Анемія

Лейкопенія

Анізоцитоз

Пойкілоцитоз

#

11

В эксперименте исследовали порог силы раздражения клеток различных тканей. Где он оказался наименьшим?

@ В мотонейронах спинного мозга

В железистых клетках

В миоцитах скелетной мышцы

В миоцитах гладкой мышцы

В кардиомиоцитах

#

12

В эксперименте на нервно-мышечном препарате лягушки изучают одиночные сокращения мышцы в ответ на электрическую стимуляцию нерва. Как изменятся сокращения мышцы после обработки препарата курареподобным веществом?

@ Исчезнут

Увеличится сила

Увеличится длительность

Уменьшится длительность

Не изменятся

#

13

В результаті травми відбулося пошкодження спинного мозку (з повним переривом) на рівні першого шийного хребця. Що відбудеться з диханням?

@ Дихання припиняється

Дихання не змінюється

Зростає частота дихання

Зростає глибина дихання

Зменшиться частота дихання

#

14

Лікар швидкої допомоги констатував у потерпілого прояви отруєння чадним газом. Яка сполука стала причиною цього?

@ Карбоксигемоглобін.

Карбгемоглобін.

Метгемоглобін.

Дезоксигемоглобін.

Оксигемоглобін.

#

15

У людини внаслідок хронічного захворювання печінки суттєво порушена її білковосинтезуюча функція. До зменшення якого параметру гомеостазу це призведе?
@ Онкотичний тиск плазми крові

Осмотичний тиск

рН

Щільність крові

Гематокрит

#

16

Длительный отрицательный эмоциональный стресс, сопровождающийся выбросом

катехоламинов, может вызвать заметное похудание. Это связано с:

@ Усилением липолиза

Нарушением пищеварения

Усилением окислительного фосфорилирования

Нарушением синтеза липидов

Усилением распада белков

#

17

У немовляти внаслідок неправильного годування виникла виражена діарея. Одним з основних наслідків діареї є екскреція великої кількості бікарбонату натрію. Яка форма порушення кислотно-лужного балансу має місце у цьому випадку?

@ Видільний ацидоз

Метаболічний алкалоз

Респіраторний ацидоз

Респіраторний алкалоз

Не буде порушень кислотно-лужного балансу

#

18

Хвора Л., 46 років скаржиться на сухість в роті, спрагу, почащений сечоспуск, загальну слабкість. При біохімічному дослідженні крові виявлено гіперглікемію, гіперкетонемію. В сечі-глюкоза, кетонові тіла. На електрокардіограмі дифузні зміни в міокарді. У хворої вірогідно:

@ Цукровий діабет

Аліментарна гіперглікемія

Гострий панкреатит

Нецукровий діабет

Ішемічна хвороба серця

#

19

На прийом до терапевта прийшов чоловік 37 років зі скаргами на періодичні інтенсивні больові приступи у суглобах великого пальця стопи та їх припухлість. При аналізі сечі встановлено її різко кислий характер і рожеве забарвлення. З наявністю яких речовин можуть бути пов’язані вказані клінічні прояви у хворого?

@ Солі сечової кислоти

Хлориди

Амонієві солі

Фосфат кальцію

Сульфат магнію

#

20

Дівчинка 10 років часто хворіє на гострі респіраторні інфекції, після яких спостерігаються множинні точкові крововиливі в місцях тертя одягу. Вкажіть, гіповітаміноз якого вітаміну має місце в дівчинки.

@ С

В6

В1

А

В2

#

21

Произошло отравление человека цианистым калием. Смерть наступила в результате образования прочного соединения цианида с:

@ Цитохромом

Рибофлавином

АТФ

ДНК

т-РНК

#

22

Після лікування хворого антибіотиками внаслідок гальмування мікрофлори кишечника можливий гіповітаміноз вітамінів:

@ В12

С

А

Р

Д

#

23

У крові хворих на цукровий діабет спостерігається підвищення вмісту вільних жирних кислот (НЕЖК). Причиною цього може бути:

@ Підвищення активності тригліцеридліпази адипоцитів

Накопичення в цитозолі пальмітоїл-КоА

Активація утилізації кетонових тіл

Активація синтезу аполіпопротеїнів А-1, А-2, А-4.

Зниження активності фосфатидилхолін-холестеин-ацилтрансферази плазми крові

#

24

У хворого К. в сечі підвищена амілазна активність і виявлено наявність трипсину, в крові підвищена амілазна активність. Про патологію якого органу це свідчить?

@ Підшлункової залози

Печінки

Шлунку

Нирок

Кишечника

#

25

У хворого виявлено зниження рН крові та вмісту бікарбонатних іонів (падіння лужного резерву крові), зростання вмісту молочної, піровиноградної кислот в крові та сечі. Який тип порушення кислотно-основної рівноваги спостерігається?

@ Метаболічний ацидоз

Респіраторний ацидоз

Метаболічний алкалоз

Респіраторний алкалоз

Дихальний алкалоз.

#

26

У реанімаційне відділення каретою швидкої допомоги доставлена жінка без свідомості. При клінічному дослідженні рівень глюкози в крові – 1,98 ммоль/л, Нв-82 г/л, еритроцити - 2,1*1012/л, ШОЕ - 18 мм/год, лейкоцити - 4,3*109/л. У хворої ймовірно:

@ Гіпоглікемія

Цукровий діабет

Галактоземія

Нестача соматотропного гормону

Нирковий діабет

#

27

При дефіциті якоговітаміну спостерігається одночасне порушення репродуктивної функції і дистрофія скелетної мускулатури?

@ Вітамін Е

Вітамін А

Вітамін К

Вітамін Д

Вітамін В1

#

28

У хворого встановлено зниження синтезу вазопресину, що призводить до поліурії і , як наслідок, до вираженої дегідратації організму. Що з переліченого є найбільш ймовірним механізмом поліурії?

@ Зниження канальцієвої реабсорбції води

Порушення канальцієвої реабсорбції іонів Nа

Зниження канальцієвої реабсорбції белка

Порушення реабсорбції глюкози

Підвищення гідростатичного тиску

#

29

Після ремонту автомобіля в гаражному приміщенні водій потрапив в лікарню з симптомами отруєння вихлопними газами. Концентрація якого гемоглобіну в крові буде підвищена?

@ Карбоксигемоглобіну

Метгемоглобіну

Карбгемоглобіну

Оксигемоглобіну

Глюкозильованного гемоглобину

#

30

У хворого, який страждає вже декілька років на карієс зубів, з’явився різкий біль в нижній щелепі, припухлість щоки, температура тіла піднялась до 37,60. Які зміни з боку крові слід очікувати у даного хворого?

@ Нейтрофільний лейкоцитоз.

Лейкопенія.

Моноцитоз.

Анемія.

Еозинофілія.

#

31

У хворого на пульпіт має місце сильно виражений набряк м'яких тканин нижньої щелепи. Який патогенетичний фактор відіграв роль у виникненні цього набряку ?

@ Підвищення проникності стінки капіляра.

Тканинна гіпоонкія

Тканинна гіпоосмія.

Кров’яна гіпоосмія.

Кров’яна гіперонкія

#

32

У відповідь на застосування знеболюючого засобу при екстракції зуба у хворого з’явились: виражений набряк м'яких тканин нижньої та верхньої щелеп, висип на шкірі обличчя, почервоніння, свербіж. Який з патологічних процесів лежить в основі такої реакції на анестетик?

@ Алергія.

Токсична дія препарату.

Запалення.

Недостатність кровообігу.

Порушення лімфовідтоку.

#

33

У стоматологічного хворого має місце сильно виражений синдром гіперсалівації (птіалізм), внаслідок чого він змушений видаляти слину з порожнини рота за допомогою серветок. Яка з форм порушення кислотно-основної рівноваги може розвинутися з часом у даного хворого?

@ Видільний ацидоз.

Газовий алкалоз..

Метаболічний ацидоз.

Видільний алкалоз.

Газовий ацидоз

#

34

У хворого, що страждає протягом багатьох років на остеомієліт нижньої щелепи, з’явилися поширені набряки, в сечі виявлена виражена масивна протеїнурія, в крові - гіперліпідемія. Яку з форм ускладнення остеомієліту найвірогідніше запідозрити у даного хворого?

@ Нефротичний синдром.

Пієліт.

Нефрит

Хронічна ниркова недостатність.

Сечо-кам’яна хвороба.

#

35

У больного внезапно наступила потеря сознания, возникли судороги. На электрокардиограмме на 2-3 зубца Р приходится 1 комплекс QRST. Какое свойство проводящей системы сердца нарушено?

@ Проводимость

Возбудимость

Автоматизм

Сократимость

-

#

36

У больного после автомобильной катастрофы АД 70/40 мм рт.ст. Больной в бессознательном состоянии. В сутки выделяет около 300 мл мочи. Каков механизм нарушения мочеобразования в данном случае?

@ Уменьшение клубочковой фильтрации

Усиление клубочковой фильтрации

Уменьшение канальцевой реабсорбции

Усиление канальцевой реабсорбции

Уменьшение канальцевой секреции

#

37

Хворий з гострим запальним процесом скаржиться на головну біль, біль у м’язах і суглобах, сонливість, гарячку. В крові встановлений лейкоцитоз, збільшення вмісту білків, у тому числі імуноглобулінів. Який з медіаторів запалення в найбільшій мірі викликає ці зміни?

@ Інтерлейкін 1.

Гістамін.

Брадикінін.

Комплемент.

Тромбоксан А2.

#

38

У больного во время приступа бронхиальной астмы при определении CO2 в крови выявлено наличие гиперкапнии [газовый ацидоз]. Какой буферной системе принадлежит решающая роль в компенсации этого состояния?

@ Гемоглобиновой

Гидрокарбонатной

Фосфатной

Белковой

Аммониогенезу

#

39

Больному для обезболивания при удалении кариозного зуба был введен раствор новокаина. Через несколько минут у него упало АД, произошла потеря сознания, возникла одышка, содороги. Какова причина возникновения анафилактического шока?

@ Сенсибилизация к новокаину

Токсическое действие новокаина

Десенсибилизация организма

Аутоаллергическое состояние

Парааллергия

#

40

У хворого 43 років спостерігається стоматит, глосит, язик малинового кольору, гладкий, В аналізі крові: Hb – 100 г/л, ер. – 2,3*1012/л, к.п. – 1,30. Чим обумовлениі стан хворого?

@ Дефіцитом вітаміну В12

Дефіцитом заліза.

Порушенням синтезу порфіринів.

Гіпоплазією червоного кісткового мозку.

Гемолізом еритроцитів.

#

41

Больному для обезболивания при удалении кариозного зуба врачом- стоматологом был введен раствор новокаина. Через несколько минут у больного появились симптомы: падение АД, учащение дыхания, потеря сознания, судороги. К какому типу аллергических реакций можно отнести это состояние?

@ Анафилактическому

Цитотоксическому

Иммунокомплексному

Замедленной гиперчувствительности

Стимулирующему

#

42

У мужчины 60 лет, обратившегося к стоматологу, был обнаружен малиновый язык и ярко-красные полосы на слизистой оболочке рта. Исследование крови у этого человека обнаружило анемию гиперхромного характера, мегалобластического типа кроветворения. Какой вид анемии был диагностирован у пациента?

@ В12-фолиево-дефицитная

Железодефицитная

Белководефицитная

Гемолитическая

Апластическая

#

43

У больного с острым пульпитом отмечается подъем температуры тела и увеличение числа лейкоцитов до 14*109/л, лейкоцитарная формула при этом: Б-0, Э-2, МЦ-0, Юн-4, Пя-8, Ся-56, Л-26, М-4. Как можно расценить такие изменения в белой крови?

@ Нейтрофилия с регенеративным сдвигом влево

Нейтрофилия с дегенеративным сдвигом влево

Нейтрофильная с гиперрегенеративным сдвигом влево

Лимфоцитоз

Нейтрофильный лейкоцитоз со сдвигом вправо

#

44

У больного наблюдается отек нижней половины лица справа, резкая пульсирующая боль в зубе, усиливающаяся при приеме горячей пищи. Врачом-стоматологом диагностирован острый пульпит. Какой механизм отека при данном заболевании является ведущим?

@ Повышение гидродинамического давления

Нарушение трофической функции нервной системы

Гиперпротеинемия

Ацидоз

Алкалоз

#

45

В експерименті К. Бернар, подразнюючи chorda tympani (гілки n. facialis) спостерігав посилення секреції піднижньощелепної слинної залози та розвиток артеріальної гіперемії. Якою за механізмом розвитку є ця гіперемія?

@ Нейротонічна

Нейропаралітична

Метаболічна

Реактивна

Робоча

#

46

У хворого 40 років у зв’язку з ураженням супраоптичних та паравентрикулярних ядер гіпоталамуса виникла поліурія (10 – 12 л за добу), полідипсія. Нестача якого гормону спричинює такі розлади?

@ Вазопресину

Окситоцину

Кортикотропіну

Соматотропіну

Тиреотропіну

#

47

У щура відтворено токсичний набряк легень за допомогою розчину хлориду амонію. Який ведучий патогенетичний фактор цього набряку?

@ Підвищення проникності капілярів

Підвищення венозного тиску

Зниження колоїдно-осмотичного тиску

Розлади нервової і гуморальної регуляції

Посилення лімфовідтоку

#

48

Піддослідному щуру внутрішньочеревно введено 10 мл 40% розчину глюкози. Через 60 хв. у щура розвинувся коматозний стан внаслідок дегідратації. Який механізм розвитку цього стану?

@ Збільшення осмотичного тиску позаклітинної рідини

Збільшення онкотичного тиску позаклітинної рідини

Зменшення синтезу вазопресину

Втрата води і солей

Порушення кислотно-основного стану

#

49

У хворого з опіками 40% поверхні тіла розвинувся опіковий шок. Який механізм розвитку шоку домінує на його початку?

@ Больовий

Зневоднення

Порушення мінерального обміну

Аутоімунізація

Порушення білкового обміну

#

50

При розгерметизації кабіни літака на висоті 19 км наступила миттєва смерть пілотів. Яка її причина?

@ Закипання крові

Крововилив в головний мозок

Газова емболія судин серця

Кровотечі

Параліч дихального центра

#

51

Чоловік на вулиці підняв двома руками електричний провід, що був під високою напругою. Наступила миттєва смерть від:

@ Фібріляції серця

Зупинки дихання

Крововилива у головний мозок

Опіків

Крововтрати

#

52

У хворого на хронічне запалення субмаксилярної слинної залози спостерігається гіпосалівація. Порушення інкреції якої речовини спостерігається при цьому?

@ Паротину

Кальцитоніну

Паратирину

Глюкагону

Соматостатину

#

53

Після отруєння фосфорорганічними речовинами у хворого виникло тривале підвищення слиновиділення. До якого порушення в організмі може призвести гіперсалівація?

@ Нейтралізації шлункового соку

Підсилення травлення у шлунку

Гіпоосмолярної дегідратації

Гіпоосмолярної гіпергідратації

Пригнічення пристінкового травлення

#

54

У хворого виявлено злоякісне новоутворення язика. Які особливості цієї пухлини дозволяють віднести її до злоякісної?

@ Інфільтративний характер росту

Експансивний характер росту

Анаплазія

Позитивний ефект Пастера

Збільшення кількості мітотичних клітин

#

55

У хворого карієс ускладнився пульпітом, що супроводжувався нестерпним болем. Яке явище при запаленні пульпи є основною причиною виникнення цього болю?

@ Ексудація

Первинна альтерація

Ішемія

Еміграція лейкоцитів

Проліферація

#

56

Хворий скаржиться на кровоточивість ясен. Який гіповітаміноз може спричинювати це явище?

@ С

В1

D

B2

A

#

57

Видалення зуба у хворого на хронічний лімфолейкоз ускладнилося тривалою кровотечею. Що може бути причиною геморагічного синдрому у цього хворого?

@ Тромбоцитопенія

Анемія

Еозинопенія

Лімфоцитоз

Нейтропенія

#

58

Видалення зуба у хворого на хронічний лімфолейкоз ускладнилося тривалою кровотечею. Назвіть провідний механізм розвитку геморагічного синдрому у цього хворого?

@ Пригнічення тромбоцитопоезу

Дефіцит вітаміну В12

Дефіцит вітаміну С

Порушення утворення колагену у судинній стінці

Порушення утворення фактору Віллебранда в судинній стінці

#

59

У хворого на остеомієліт верхньої щелепи спостерігається підвищення температури тіла протягом доби до 400С, що різко знижується до 35,60С. Для якого типу температурної кривої це характерне?

@ Гектичного [hectica]

Постійного [соntinua]

Інтермітуючого [intermittens]

Поворотного [reccurens]

Атипового [atypica]

#

60

Після введення пірогеналу у людини спостерігається блідість шкіри, озноб, "гусяча шкіра", при визначені газообміну – збільшення споживання кисню. Для якої стадії гарячки найбільш характерні такі зміни.

@ підвищення температури

стояння температури на підвищеному рівні

зниження температури шляхом кризису.

зниження температури шляхом лізису.

-

#

61

Ліквідатор наслідків аварії на ЧАЕС отримав дозу іонізуючого опромінення 6 Гр. Які зміни лейкоцитарної формули слід очікувати через 10 днів?

@ Агранулоцитоз

Лімфоцитоз

Лейкоцитоз з лімфоцитопенією

Базофілію

Єозінофілію

#

62

У дитини 3-х місяців на ділянці вух, носа, щік та на склерах з'явилися темні плями, а сеча при стояні на повітрі ставала чорною. Накопичення якої сполуки слід очікувати в крові та сечі?

@ Гомогентизинової кислоти

Фенілаланіну

Меланіну

Триптофану

Галактози

#

63

У ликвидатора последствий аварии на ЧАЭС, получившего дозу облучения 5Гр, через неделю при анализе крови определено наличие агранулоцитоза. Какой патогенетический механизм является ведущим в его возникновении?

@ Угнетение лейкопоэза

Увеличенный переход гранулоцитов в ткани

Увеличение разрушения лейкоцитов

Нарушение выхода зрелых лейкоцитов из костного мозга

Развитие аутоиммунного процесса

#

64

Судебно-медицинский эксперт при вскрытии трупа 20-летней девушки установил наступление смерти вследствие отравления цианидами. Нарушение какого процесса наиболее вероятно было причиной смерти девушки?

@ Тканевого дыхания

Транспорта кислорода гемоглобином

Синтеза гемоглобина

Синтеза гликогена

Окислительного фосфорилирования

#

65

В группе детей, которые ели арбуз, у одного из них появились: слабость, головокружение, рвота, одышка, тахикардия, акроцианоз. Лабораторный анализ арбуза показал высокое содержание нитратов. Какой ведущий механизм в патогенезе отравления у этого ребенка?

@ Недостаточность мет Нв-редуктазы

Недостаточность супероксиддисмутазы

Блокада цитохромоксидазы

Недостаточность глутатион-перокси дазы

Недостаточность каталазы
66

У мужчины 35 лет через 15 минут после автомобильной аварии выявлена массивная травма нижних конечностей без значительной внешней кровопотери. Пострадавший пребывает в возбужденном состоянии. Какой компонент патогенеза травматического шока является у пациента ведущим и требует немедленной коррекции?

@ Боль

Острая почечная недостаточность

Интоксикация

Нарушение функции сердца

Внутренняя плазмопотеря

#

67

Хворий, 52 роки, був діставлен в клініку з симптомами гострого панкреатиту та явищами панкреатичного шоку. Що є основним механізмом розвитку панкреатичного шоку?

@ Надходження активованих протеаз залози в кров

Стимулювання панкреатичної секреції

Порушення відтоку панкреатичного соку

Зменшення інкреції інсуліну

Зниження артеріального тиску

#

68

У 37-річної хворої в аналізі крові : Нв – 60 г/л, еритроцити – 3,0*1012/л, колірний показник – 0,6. Лейкоцитарна формула без змін, тромбоцити – 200*109/л, ретикулоцити – 0,3%, ШОЕ – 18 мм/час. Мікроцитоз, пойкілоцитоз ерітроцитів. Яка це анемія за механізмом розвитку?

@ Залізодефіцитна

Гостра постгеморагічна

Гемолітична

В12 – фолієводефіцитна

Гіпопластична

#

69

Чоловіку 37 років при лікуванні гострого пульпіту було введено розчин новокаіну. Через кілька хвилин у пацієнта розвився анафілактичній шок. З яким імуноглобуліном головним чином взаємодіє в організмі антиген при даній алергічній реакції?

@ IqE

IqM

IqA

IqD

IqG

#

70

Больной обратился с жалобами на боли саднящего характера на верхнем небе, затрудненное глотание. В последнее время появилась общая слабость, потерял в весе. При обследовании был диагностирован рак слизистой оболочки рта с метастазами в лимфоузлы. Каков механизм развития кахексии у данного больного?

@ Снижение пластических и энергетических резервов

Нарушение желудочной секреции

Нарушение трофической функции нервной системы

Нарушение функции эндокринной системы

Усиление гликонеогенеза

#

71

У больного, которому был экстирпирован зуб по поводу острого гнойного периостита, отмечалось длительное, неостановимое обычными методами кровотечение из лунки. Анализ крови у этого больного дал следующие изменения: Эр-2,9*1012/л, Нв- 90г/л; Ц.П.- 0,9; Тромбоц.- 60*109/л; Лейкоц- 52*109/л. Б-0, Э-1, Мц-0, Юн-0, Пя-2, Ся-18, Л-8, М-1, Миелобластов-70. Какое заболевание крови наблюдается у этого больного?

@ Острый миелоидный лейкоз

Хронический миелоидный лейкоз

Недифференцируемый лейкоз

Эритромиелоз

Промиелоцитарный лейкоз

#

72

У пациента 50 лет, обратившегося к стоматологу был обнаружен малиновый, "лакированный" язык. При обследовании в крови: снижено количество эритроцитов и концентрации гемоглобина, цветовой показатель 1,3, появились признаки мегалобластического типа кроветворения, дегенеративные изменения в белой крови. Какое заболевание крови было обнаружено у больного?

@ В12-фолиеводефицитная анемия

Железодефицитная анемия

Миелоидный лейкоз

Апластическая анемия

Гемолитическая анемия

#

73

У хворого на хронічний мієлолейкоз виникли ознаки виразково-некротичного стоматиту. При біопсії слизової оболонки виявлені лейкозні клітини. З якою ланкої патогенезу пухлини пов’язане ураження ротової порожнини ?

@ пухлинна прогресія

Мутаційний механізм трансформації

Епігеномний механізм трансформації

Промоція

Ініціація

#

74

У хворого на глосит спостерігається зникнення сосочків на язиці, його почервоніння та печія. При аналізі крові встановлено: кількість еритроцитів – 2,2*1012/л, гемоглобін – 103 г/л кольоровий показник – 1.4. Яка анемія спостерігається у цього хворого?

@ В12-фолієводефіцитна

Залізодефіцитна

Альфа-таласемія

Бета-таласемія

Залізо-рефрактерна

#

75

Кровотечу, що виникла у дитини після видалення зуба, не вдавалося припинити протягом 6 годин. Проведене дослідження системи гемостазу встановило різке зменшення вмісту VIII фактору зсідання крові. За яким типом спадкується це захворювання?

@ Зчеплене із статевою хромосомою

Аутосомно-домінантний

Аутосомно-рецисивний

Полігенний

Неповне домінування

#

76

У хворого на хронічний гепатит видалили зуб. Кровотечу, що виникла після цього не вдавалося припинити протягом 2 годин. Проведене дослідження системи гемостазу встановило зменшення вмісту декількох факторів зсідання крові. Порушенння якої функції печінки призвело до порушення гемостазу у цього хворого?

@ Білоксинтезуюча

Антитоксична

Травна

Гормональна

Захисна

#

77

Жінка із токсикозом вагітності страждає на гіперсалівацію, що призводить до втрати 3-4 літрів слини щоденно. Яке порушення водно-сольового обміну виникає при цьому?

@ Гіпогідратація гіперосмолярна

Гіпогідратація гіпосмолярна

Гіпогідратація ізоосмолярна

Гіпокаліємія

Гіпонатріємія

#

78

При повному (з водою) аліментарному голодуванні розвинулись генералізовані набряки. Який із патогенетичних факторів у цьому випадку є ведучим?

@ Зниження онкотичного тиску плазми крові.

Зниження гідростатичного тиску міжклітинної рідини.

Зниження осмотичного тиску плазми крові.

Підвищення онкотичного тиску тканинної рідини.

Підвищення осмотичного тиску міжклітинної рідини.

#

79

Опікова хвороба, окрім іншого, характеризується розвитком анемії, однією з причин якої вважається дефект:

@ Еритропоетину

Мієлопоетину

Вітаміну В12

Катехоламінів

Тромбопоетину

#

80

У больного С., выявлены такие изменения в периферической крови: Эр. 3,2*1012/л, Гем. 80 г/л, Лейк. 25*109/л. Лейкоцитарная формула: базофилы - 5%, эозинофилы - 9%, миелобласты - 3%, промиелоциты - 8%; миелоциты - 11%, метамиелоциты - 22%, палочкоядерные - 17%, сегментоядерные - 19%, лимфоциты - 3%, моноциты - 3%. Определите наиболее вероятную патологию соответствующую данному описанию картины крови:

@ хронический миелолейкоз

острый миелобластный лейкоз

эритромиелоз

лейкемоидная реакция

недеферинцируемый лейкоз

#

81

У больного П., выявлены такие изменения в периферической крови: Эр. 3,0*1012/л, Гем. 80 г/л, Лейк. 21*109/л. Лейкоцитарная формула: базофилы - 0%, эозинофилы - 0%, миелобласты - 54%, промиелоциты - 1%; миелоциты - 0%, метамиелоциты - 0%, палочкоядерные - 1%, сегментоядерные - 28%, лимфоциты - 13%, моноциты - 3%. Определите наиболее вероятную патологию соответствующую данному описанию картины крови:

@ острый миелобластный лейкоз

хронический миелолейкоз

эритромиелоз

лейкемоидная реакция

недеферинцированный лейкоз

#

82

Больной В. 38 лет, доставлен в приемное отделение с признаками гипоксии развившейся после отравления угарным газом. Состояние средней тяжести, тахикардия, одышка, АД 160/100. Какой механизм токсического действия окиси углерода на организм?

@ Образование карбоксигемоглобина

Образование метгемоглобина

Нарушение диссоциации оксигемоглобина

Образование карбгемоглобина

Блокада кальциевых каналов эритроцитов

#

83

У больного П., после травмы возникла необходимость введения противостолбнячной сыворотки, однако проба на чувствительность к сыворотке оказалась положительной. Как провести специфическую гипосенсибилизацию у больного? Введением:

@ малых дробных доз специфического аллергена

физиологических доз глюкокортикоидов

разрешающей дозы специфического аллергена

лечебных доз антигистаминных препаратов

наркотических веществ снижающих чувствительность

#

84

Больная 23 лет жалуется на выраженную слабость, сонливость, потемнение в глазах, головокружение, извращение вкуса. В анализе меноррагии. Объективно: бледность кожных покровов, трещины в углах рта, слоящиеся ногти, увеличение ЧД и ЧСС. Анализ крови: Эр 2,8*1012/л , Hb 70 г/л, ЦП - 0,75. Какая гипоксия вероятнее всего привела к развитию выявленных симптомов у больной?

@ Гемическая

Циркуляторная

Тканевая

Респираторная

Субстратная

#

85

У больной 43-х лет, на фоне септического шока отмечается тромбоцитопения, уменьшение фибриногена, появление в крови продуктов деградации фибрина, появление петехиальных кровоизлияний. Укажите причину возникновения данных изменений.

@ ДВС-синдром

Аутоиммунная тромбоцитопения

Геморрагический диатез

Нарушение выработки тромбоцитов

Экзогенная интоксикация

#

86

У женщины на 7-м месяце беременности стала быстро нарастать анемия: Эритроциты - 2,7*1012/л, Нв -110 г/л, ЦП -1,2 анизоцитоз, пойкилоцитоз, единичные мегалоциты. Какой вид анемии развился у женщины?

@ В12 - дефицитная анемия

Железодефицитная анемия

Гемолитическая анемия

Постгеморрагическая анемия

Таласемия

#

87

У беременной женщины развился токсикоз с тяжелыми повторными рвотами на протяжении суток. К концу суток начали проявляться тетанические судороги и обезвоживание организма. Какой сдвиг КЩР вызвал описанные изменения?

@ Негазовый выделительный алкалоз

Газовый алкалоз

Газовый ацидоз

Негазовый метаболический ацидоз

Негазовый выделительный ацидоз

#

88

Во время обеда ребенок поперхнулся и аспирировал пищу. Начался сильный кашель, кожа и слизистые цианотичны, пульс учащен, дыхание редкое, Выдох удлинен. Какое нарушение внешнего дыхания развилось у ребенка?

@ Стадия экспираторной одышки при асфиксии

Стадия инспираторной одышки при асфиксии

Дыхание Биота

Дыхание Куссмауля

Дыхание Чейна-Стокса

#

89

При підйомі в гори у альпініста розвинулась ейфорія, головний біль, запаморочення, серцебиття, задишка, яка чергувалася з апное. Яке порушення кислотно-основного стану розвинулося у альпініста?

@ Газовий алкалоз

Метаболічний алкалоз

Негазовий алкалоз

Газовий ацидоз

Негазовий ацидоз

#

90

Внаслідок поранення хворий втратив 25% об’єму циркулюючої крові. Назвіть терміновий механізм компенсації крововтрати.

@ Находження міжтканинної рідини в судини

Відновлення білкового складу крові

Збільшення числа ретикулоцитів

Відновлення числа еритроцитів

Активація еритропоезу

#

91

У робітника, який працював літом у щільному костюмі, різко підвищилась температура тіла, з’явились задишка, тахікардія, нудота, судоми, втрата свідомості. Що явилось причиною тяжкого стану робітника?

@ Зниження тепловіддачі

Підвищення теплопродукції

Підвищення тепловіддачі

Зниження теплопродукції

Тепловіддача дорівнює теплопродукції

#

92

В приймальне відділення поступила дитина 1,5 р. з ознаками отруєння нітратами: стійкий ціаноз, задишка, судоми. Утворення якої форми гемоглобину лежить в основі цих симптомів?

@ метгемоглобіну.

карбгемоглобіну

карбоксигемоглобіну

редукованого гемоглобіну

оксигемоглобіну

#

93

Хвора поступила в клініку на обстеження. З дитинства відмічалось зниження гемоглобіну до 90-95 г/л. Лікування препаратами заліза було неефективне. Аналіз крові при поступленні: Е– 3,2*1012/л, Hb– 85 г/л, к.п.– 0,78. В мазку анізоцитоз, пойкілоцитоз, мішеневидні еритроцити, ретикулоцити –16%. Поставлений діагноз – таласемія. До якого виду гемолітичних анемій можна віднести дане захворювання?

@ Спадкова гемоглобінопатія.

Спадкова мембранопатія

Набута мембранопатія

Спадкова ферментопатія

Набута ферментопатія

#

94

Хворий на протязі останнього року став відмічати під_вищену втомлюваність, загальну слабість. Аналіз крові: Е– 4,1*1012/л, Hb– 119г/л, к.п.– 0.87, лейкоцити – 57*109/л, лейкоформула: Ю–0, П–0, С–9%, Е–0, Б–0, лімфобласти–2%, пролімфоцити–5%, лімфоцити–81%, М–3%, тромбоцити–160*109/л. В мазку: нормохромія, велика кількість тіней Боткіна–Гумпрехта. Про яку патологію системи крові свідчить дана гемограма?

@ Хронічний лімфолейкоз.

Хронічний мієлолейкоз

Гострий лімфобластний лейкоз

Гострий мієлобластний лейкоз

Хронічний монолейкоз

#

95

Масугі викликав розвиток гломерулонефриту у щурів таким чином: гомогенат нирок щура вводив кролю. Через декілька тижнів сироватку сенсибілізованого кроля вводив щурам. Який тип алергічної реакциї за Джеллом та Кумбсом лежить в основе розвитку гломерулонефрита у щурів?

@ Цитотоксичний

Анафілактичний

Імунокомплексний

Гіперчутливість сповільненого типу

Стимулюючий

#

96

У хворого виявлено порушення прохідності дихальних шляхів на рівні дрібних і середніх бронхів. Які порушення кислотно–лужної рівноваги можна виявити у крові в даному випадку?

@ Респіраторний ацидоз.

Метаболічний ацидоз

Респіраторний алкалоз

Метаболічний алкалоз

-

#

97

До неврологічного відділення з приводу мозкового крововиливу надійшов хворий 62-х років. Стан важкий. Спостерігається наростання глибини та частоти дихання, а потім його зменшення до апное, після чого цикл дихальних рухів відновлюється. Який тип дихання виник у хворого?

@ Чейна–Стокса

Кусмауля

Біота

Гаспінг-дихання

Апнеїстичне

#

98

У чоловіка 52 років через 3 роки після операції видалення шлунку вміст еритроцитів в крові складає 2,0*1012/л, Hb– 85г/л, к.п.– 1,27. Порушення засвоєння якого вітаміну викликало такї зміни?

@ B12.

B6

C

P

A

#

99

На шостому місяці вагітності в жінки з’явилася виражена залізодефіцитна анемія. Діагностичною ознакою її була поява в крові

@ Анулоцитів

Макроцитів

Пойкілоцитів

Ретикулоцитів

Нормоцитів

#

100

У хворого 38 років, який переніс гепатит і продовжував вживати алкоголь, розвинулися ознаки цирозу печінки з асцитом і набряками на нижніх кінцівках. Які зміни складу крові стали вирішальними в розвитку набряків?

@ Гіпоальбумінемія

Гіпоглобулінемія

Гіпохолестеринемія

Гіпокаліємія

Гіпоглікемія

#

101

Хворому з закритим переломом плечевої кістки накладена гіпсова пов’язка. Наступного дня з’явилася припухлість, синюшність і похолодання кисті травмованої руки. Про який розлад периферичного кровообігу свідчать ці ознаки?

@ Венозна гіперемія

Артеріальна гіперемія

Ішемія

Тромбоз

Емболія

#

102

Після вимушеного швидкого підняття водолаза з глибини на поверхню у нього з’явилися ознаки кесонної хвороби – біль у суглобах, свербіння шкіри, мерехтіння в очах, затьмарення свідомості. Яким видом ємболії вони були зумовлені?

@ Газовою

Повітряною

Жировою

Тканинною

Тромбоємболією

#

103

У хворого Н. приступи гарячки виникають через день. Під час приступу температура різко підвищується і утримується на високому рівні до 2 год, а потім знижується до вихідного рівня. Цей тип гарячки характерний для

@ Малярії

Поворотного тифу

Сепсису

Бруцельозу

Висипного тифу

#

104

В лікарню доставлений непритомний чоловік після отруєння чадним газом. Гіпоксія у нього зумовлена появою у крові

@ Карбоксигемоглобіну

Метгемоглобіну

Карбгемоглобіну

Оксигемоглобіну

Дезоксигемоглобіну

#

105

Після занурення водолаза на глибину 60 м у нього з’явилися симптоми порушення функцій центральної нервової системи – збудження, ейфорія, ослаблення уваги, професійні помилки. Ці симптоми пов’язані з токсичною дією на нейрони

@ Азоту

Кисню

Вуглекислого газу

Аміаку

Лактату

#

106

До дерматолога звернулася пацієнтка із скаргами на екзематозне ураження шкіри рук, що з’являється після контакту з миючим засобом “Лотос”. Використання гумових рукавичок запобігає цьому. Патологічна реакція шкіри зумовлена активацією

@ Т-лімфоцитів

В-лімфоцитів

Моноцитів

Нейтрофілів

Базофілів

#

107

У дитини двох років встановлено діагноз гіпоплазії тимуса. Який показник стану імунної системи є найбільш характерним для цього імунодефіцита?

@ Зниження кількості Т-лімфоцитів

Зниження кількості В-лімфоцитів

Дефіцит Т и В-лімфоцитів

Відсутність плазматичних клітин

Зниження іммуноглобулінів М

#

108

У хворого на пневмонію виникла гарячка. Що безпосередньо спричинює зміну установочної точки температури в нейронах гіпоталамуса цього хворого?

@ Простагландини Е1, Е2

Ендотоксин

Екзотоксин

Інтерлейкін-2

Тромбоцитарний фактор росту

#

109

У больного в третьем периоде лихорадки наступило критическое падение температуры тела. При этом наблюдалась тахикардия и снижение артериального давления до 80/60 мм рт.ст. Какой вид коллапса развился при этом?

@ Инфекционно- токсический

Ортостатический

Геморрагический

Кардиогенный

Панкреатический

#

110

У больного, прооперированного по поводу осложненного аппендицита, в анализе крови отмечаются следующие изменения: Эр.- 4,0*1012/л, Нв - 120г/л, Ц.п.-0,9, Лейкоц. - 18*109/л, Б - 0, Э - 0, Мц - 0, Юн - 0, Пя - 20. Ся - 53, Л -21, М - 5. Как называются такой ядерный сдвиг лейкоцитарной формулы?

@ Дегенеративный сдвиг влево

Сдвиг вправо

Регенеративный сдвиг влево

Гиперрегенеративный

Регенеративно-дегенеративный
111

На ранній стадії цукрового діабету у хворих спостерігається поліурія. Чим вона зумовлена?

@ Гіперглікемія

Кетонемія

Гіпохолестеринемія

Гіперхолестеринемія

Гіперкалійемія

#

112

У хворого 35 років розвинулася імунна гемолітична анемія. Який показник сироватки крові зросте в найбільшій мірі?

@ Непрямий білірубін

Прямий білірубін

Стеркобіліноген

Мезобіліноген

Протопорфірин

#

113

У хворих на поворотний тиф виникає лихоманка, яка характеризується кількаденними періодами високої гарячки, що чергується з періодами нормальної температури. Така температурна крива називається:

@ Febris recurrns

Febris hectica

Febris intermittens

Febris continua

Febris atypica

#

114

У хворого виявлено порушення прохідності дихальних шляхів на рівні дрібних і середніх бронхів. Які зміни кислотно-основної рівноваги можуть розвинутись у пацієнта?

@ Респіраторний ацидоз

Респіраторний алкалоз

Метаболічний ацидоз

Метаболічний алкалоз

КОР не зміниться

#

115

У дитини, хворої на дифтерію, розвинувся набряк гортані. Який розлад дихання спостерігається у даному випадку?

@ Диспное (задишка)

Гаспінг-дихання

Апнейстичне дихання

Дихання Куссмауля

Дихання Біота

#

116

У больного с инфекционным заболеванием температура тела через сутки повышалась до 39,5-40,50С и держалась на этой высоте около часа, а затем возвращалась к исходному уровню. Какой тип лихорадочной кривой описан в данном случае?

@ Перемежающаяся

Постоянная

Послабляющая

Изнуряющая

Атипичная

#

117

Пострадавшему в ДТП наложили кровоостанавливающий жгут на верхнюю треть бедра на 3 часа. После снятия жгута у больного появился резкий отек ткани бедра, частый пульс, холодный пот, резкая гипотония. Какой патологический процесс развился у больного?

@ Токсемический шок

Анафилактический шок

Коллапс

Кардиогенный шок

Геморрагический шок

#

118

Хворий 23 років поступив у лікарню із черепно-мозковою травмою у важкому стані. Дихання характеризується судомним тривалим вдохом який переривається коротким видихом. Для якого типу дихання це характерно?

@ Апнейстичного

Гаспінг-дихання

Куссмауля

Чейн - Стокса

Біота

#

119

У больного Б., на 2-е сутки после развития инфаркта миокарда произошло резкое падение систолического АД до 60 мм.рт.ст. с тахикардией 140 уд/мин, одышкой, потерей сознания. Какой механизм является ведущим в патогенезе развившегося шока?

@ Уменьшение ударного объема крови

Интоксикация продуктами некротического распада

Снижение объёма циркулирующей крови

Пароксизмальная тахикардия

Анафилактической реакции на миокардиальные белки

#

120

У 48 летнего пациента после сильной психоэмоциональной нагрузки внезапно появилась острая боль в области сердца с иррадиацией в левую руку. Нитроглицерин снял приступ боли через 10 минут. Какой патогенетический механизм является ведущим в развитии этого процесса?

@ Спазм коронарных сосудов

Расширение периферических сосудов

Закупорка коронарных сосудов

Сдавление коронарных сосудов

Повышение потребностей миокарда в кислороде

#

121

У хлопчика 3 років з вираженим геморагічним синдромом відсутній антигемофільний глобулін А (фактор VIII) в плазмі крові. Яка фаза гемостазу первинно порушена у цього хворого?

@ Внутрішній механізм активації протромбінази

Зовнішній механізм активації протромбінази

Перетворення протромбіну в тромбін

Перетворення фібриногену в фібрин

Ретракція кров’яного згустку

#

122

У хворого, 42 років, скарги на болі в епігастральній ділянці, блювоту; блювотні маси кольору “кофейної гущі”; мелена. В анамнезі виразкова хвороба шлунку. Аналіз крові: еритроцити – 2,8*1012/л , лейкоцити – 8*109/л, гемоглобін 90 г/л. Вкажіть найбільш ймовірне ускладнення, яке виникло у хворого?

@ Кровотеча

Пенетрація

Перфорація

Пререродження в рак

Пілоростеноз

#

123

У хворого на цукровий діабет розвинулася діабетична кома внаслідок порушення кислотно-основного стану. Який вид порушення виник при цьому?

@ Метаболічний ацидоз

Метаболічний алкалоз

Респіраторний ацидоз

Газовий алкалоз

Негазовий алкалоз

#

124

Хворому віком 45 років встановлено діагноз: виразкова хвороба шлунку. При дослідженні секреторної функції шлунку було виявлено, що кількість базального секрету складає 100 мл/год., а кислотність базального секрету 60 ммоль/л. Дія якого фактора сприяє гіперсекреції в шлунку?

@ Гастрин

Глюкагон

Соматостатин

Панкреатичний поліпептид

Бета-ендорфін

#

125

При работе с радиоактивными веществами сотрудник вследствие аварии получил дозу общего облучения 4 Гр. Жалуется на головную боль, тошноту, головокружение. Какие изменения в составе крови можно ожидать у больного через 10 часов после облучения?

@ Нейтрофильный лейкоцитоз

Лимфоцитоз

Лейкопению

Агранулоцитоз

Нейтропению

#

126

Больному 25 лет установлен диагноз хронического гепатита. Больной жалуется на потерю массы тела на 10 кг на протяжении 2 месяцев. Объективно: кожа сухая, шелушащаяся, бледная с желтоватым оттенком, мелкоточечные кровоизлияния на коже, кровоточивость десен. Нарушение какой функции печени отражают мелкоточечные кровоизлияния и кровоточивость десен:

@ Белковосинтетической

Пигментообразующей

Гликогенсинтетической

Детоксицирующей

Депонирующей

#

127

У тварини викликали карієс. Який компенсаторний механізм є найважливішим при розвитку цього захворювання?

@ Утворення одонтобластами вторинного дентину

Новоутворення емалі

Гіпотрофія слинних залоз

Пригнічення фагоцитозу

Гіперфункція прищитовидних залоз

#

128

У хворого на артеріальну гіпертензію наслідком гіпертонічного кризу стала гостра серцева недостатність. Який механізм серцевої недостатності є головним в даному випадку?

@ Перевантаження серця опором

Перевантаження серця припливом крові

Ушкодження міокарда

Абсолютна коронарна недостатність

Відносна коронарна недостатність

#

129

Чоловік 20 років, що приймав участь у ліквідації наслідків Чорнобильської катастрофи захворів на пародонтоз. Який етіологічний фактор є найбільш важливим у розвитку цієї патології?

@ Стрес

Дефіцит заліза

Неповноцінне харчування

Підвищення фізичного навантаження на зубощелепний аппарат

Стрептококки порожнини рота

#

130

У хворого на запалення трійчастого нерва останніми роками прогресує пародонтоз. Який фактор є головним у розвитку пародонтозу?

@ Нейродистрофічні порушення в пародонті

Зменшення активності лейкоцитарної еластази

Послаблення утворення імуноглобулінів

Підвищення тонуса вагуса

Зниження активності калікреїн-кінінової системи

#

131

У чоловіка 35-ти років через 30 хвилин після автомобільної аварії виявлена масивна травма нижніх кінцівок без значної зовнішньої крововтрати. Постраждалий знаходиться в збудженному стані. Який компонент патогенезу травматичного шоку є в пацієнта ведучим та потребує негайного корегування?

@ Біль

Внутрішня крововтрата

Внутрішня плазмовтрата

Інтоксікація

Порушення функції органів

#

132

У хворого на променеву хворобу з’явилися ознаки геморагічного синдрому. Який механізм має найбільше значення у патогенезі цього синдрому?

@ Тромбоцитопенія

Лімфопенія

Еритропенія

Еозинопенія

Нейтропенія

#

133

Больной 56 лет, страдает фиброзно-кавернозным туберкулезом легких. В течение последних 3 недель усилился кашель, слабость, увеличилось количество отделяемой гнойно-слизистой мокроты с прожилками крови. Какова причина возникшей вентиляционной недостаточности в данном случае?

@ Уменьшение количества функционирующих альвеол

Нарушение функции дыхательного центра

Нарушение функции нервно-мышечного аппарата

Нарушение подвижности грудной клетки

Нарушение проходимости воздухоносных путей

#

134

Больной 68 лет перенес инфаркт миокарда. При ЭКГ - обследовании отмечается прогрессирующее увеличение интервала PQ вплоть до выпадения комплекса QRS, после чего интервал PQ восстанавливается. С нарушением какой функции сердца связанно данное нарушение сердечного ритма?

@ Проводимость

Возбудимость

Сократимость

-

Автоматизм

#

135

У больного 37 лет, после отравления неизвестным лекарственным препаратом, отмечаются быстрые непроизвольное стереотипные сокращения мускулатуры лица, имитирующие мигание и прищуривание. К какой форме расстройств двигательной функции нервной системы следует отнести описанные нарушения?

@ Гиперкинез

Гипокинез

Акинезия

Атаксия

-

#

136

У підлітка було видалено зуб з застосуванням новокаїну. Через 10 хвилин у нього з явились блідість шкірних покривів, задишка, розвилася гіпотензія. Який тип алергічної реакції виник у підлітка?

@ Анафілактична

Цитотоксична

Імунокомплексна

Клітинно-опосредкована

Стимулююча

#

137

У хворої ушкодження задньої долі гіпофізу призвело до збільшення добового діурезу до 10-15 л. Що є головним механізмом у розвитку поліурії?

@ Дефіцит вазопресину

Надлишок вазопресину

Надлишок альдостерону

Надлишок натрійуретичного фактора

Дефіцит кортикотропіну

#

138

У хворого, що тривалий час страждає на хронічний гломерулонефрит, виникла азотемія, олігурія, гіпо-, ізостенурія, протеїнурія. Що є головною ланкою в патогенезі цих симптомів при хронічній нирковій недостатності?

@ Зменшення маси діючих нефронів

Підвищення клубочкової фільтрації

Зниження канальцевої секреції

Порушення проникності клубочкових мембран

Посилення реабсорбції натрію

#

139

У жінки після пологів зменшилася маса тіла на 20 кг, випадають зуби та волосся, спостерігається атрофія м’язів [гіпофізарна кахексія]. З порушенням синтезу якого гормону гіпофізу це пов’язано?

@ Соматотропного

Кортикотропного

Тиреотропного

Гонадотропного

Пролактину

#

140

У хворої жінки 29-ти років місяцеподібне обличчя, ожиріння верхньої частини тулуба, стриї на передній черевній стінці, гірсутизм, у сечі підвищено рівень 17-оксікетостероїдів. Вищезазначені прояви характерні для:

@ Синдрому Іценко-Кушинга

Феохромоцитоми

Синдрому Кона

Первинного гіперальдостеронізму

Вторинного гіперальдостеронізму

#

141

Хлопчик В. 12 років , повернувшись зі школи , почав скаржитись на головний біль , нудоту , знобіння , періодичні болі у м’язах , втрату апетиту , стомлюваність . Для якого періоду хвороби характерні такі симптоми ?

@ Продромального

Латентного

Инкубаційного

Розпалу захворювання

Закінчення хвороби

#

142

У жінки віком 45 років, яка тривалий час хворіє на бронхіальну астму, виник напад ядухи. Який патогенетичний механізм має це явище?

@ Спазм дрібних бронхів.

Втрата еластичності легеневої тканини.

Зниження чутливості дихального центру.

Порушення рухомості грудної клітини.

Порушення перфузії легеневої тканини .

#

143

У хворого має місце передсердно-шлуночкова блокада І ступеня, що супроводжується подовженням інтервалу P-Q до 0,25 с. Порушення якої функції серця має місце за цих умов?

@ Провідності

Автоматизму

Збудливості

Скоротливості

Засвоєння ритму

#

144

В параллельных експериментах на крысах, которые подвергались длительному прямому солнечному облучению, и в камерах, закрытых стеклом, было отмечено возникновение опухолей непокрытых шерстью частей кожи у животных, которые находились в открытых камерах. С влиянием какого из перечисленнях ниже факторов связано это явление?

@ Ультрафиолетового облучения

Эндогенных химических канцерогенов

Биологических канцерогенов

Экзогенных химических канцерогенов

Инфракрасного облучения

#

145

В отделение реанимации доставлен больной, в крови которого обнаружено высокое содержание сульфгемоглобина. Какой тип гипоксии имеет место в данном случае?

@ Гемический

Респираторный

Циркуляторный

Тканевой

Экзогенный

#

146

При воспалении в сосудах микроциркуляторного русла отмечается повышение их проницаемости, увеличение гидродинамического давления. В межтканевой жидкости имеет место повышение осмотической концентрации и дисперсности белковых структур. Какой вид отека будет наблюдаться в данном случае?

@ Смешанный

Гидродинамический

Коллоидно-осмотический

Лимфогенный

Мембранногенный

#

147

При подъеме в горы на высоте 5000 метров у участников альпинистской группы появились жалобы на одышку, учащенное сердцебиение, головную боль, головокружение, звон в ушах. Какой патогенетический фактор определяет указанные явления?

@ Гипоксемия

Гипокалиемия

Увеличение кислородной емкости крови

Лактацидемия

Гипернатриемия

#

148

У медсестры манипуляционного кабинета со стажем работы 20 лет развился контактный дерматит верхних конечностей. К какому типу иммунных нарушений относится данное заболевание?

@ Аллергическая реакция замедленного типа

Первичный иммунодефицит

Аллергическая реакция немедленного типа

В-клеточный иммунодефицит

Т-клеточный иммунодефицит

#

149

Пациент 16 лет, страдающий болезнью Иценко-Кушинга, обратился по поводу избыточного веса тела. При опросе выяснилось, что энергетическая ценность потребляемой пищи составляет 1700-1900 ккал/сут. Какова ведущая причина ожирения в данном случае?

@ Избыток глюкокортикоидов

Недостаток инсулина

Избыток инсулина

Недостаток глюкокортикоидов

Гиподинамия

#

150

Після введення місцевого анестетика у пацієнта розвинувся анафілактичний шок. Який механізм порушення кровообігу є провідним при цьому?

@ Зменшення тонусу судин

Гіперволемія

Біль

Активація симпато-адреналової системи

Зниження скоротливої функції серця

#

151

У людини, яку покусали бджоли, розвинувся набряк верхніх кінцівок та обличчя. Який основний патогенетичний механізм розвитку цього набряку?

@ Підвищення проникності стінки судин

Збільшення гідростатичного тиску в капілярах

Зменшення гідростатичного тиску тканин

Збільшення онкотичного тиску тканин

Зменшення онкотичного тиску крові

#

152

У хворого діагностовано хронічний гломерулонефрит. Внаслідок значних склеротичних змін маса функціонуючих нефронів зменшилася до 10\%. Яке з перерахованих нижче порушень лежить в основі наростаючого уремічного синдрому?

@ Азотемія

Порушення водного гемостазу

Порушення осмотичного гемостазу

Ниркова остеодистрофія

Артеріальна гіпертензія

#

153

Після нападу бронхіальної астми хворому проведено дослідження периферичної крові. Які зміни очікуються?

@ Еозинофілія

Лейкопенія

Лімфоцитоз

Тромбоцитопенія

Еритроцитоз

#

154

У результаті пошкодження одного з реакторів АЕС пройшло витікання радіоактивних продуктів. Люди, які знаходилися в зоні підвищеної радіації, орієнтовно отримали по 250-300 Р. Їх негайно госпіталізовано. Яка зміна крові буде характерна у цей період?

@ Лімфопенія

Лейкопенія

Анемія

Тромбоцитопенія

Нейтропенія

#

155

Через рік після субтотальної резекції шлунку з приводу виразки малої кривизни виявлені зміни в аналізі крові - анемія, лейко- та тромбоцитопенія, КП- 1,3, наявність мегалобластів та мегалоцитів. Дефіціт якого фактору обумовив розвиток цієї патології?

@ Фактора Касла

Хлороводневої кислоти

Муцину

Пепсину

Гастрину

#

156

Больная 55-ти лет с острым приступом печеночной колики поступила в гастроэнтерологическое отделение. Объективно: температура тела 38 0С, склеры, слизистые и кожа иктеричны, моча темная, кал светлый. Жалобы на кожный зуд. Какая причина желтухи у данной больной?

@ обтурация желчных ходов

деструкция гепатоцитов

усиленный распад эритроцитов

нарушение липидного обмена

длительный прием продуктов богатых каротином

#

157

У больной, страдающей гипертиреозом, повышена температура тела. Каков механизм повышения температуры тела?

@ Разобщение окислительного фосфорилирования в митохондриях

Повышение утилизации глюкозы тканями

Усиление гликогенолиза

Усиление катаболизма белков

Ускорение окисления жира в печени

#

158

Пострадавший 45-ти лет доставлен бригадой скорой помощи с тяжелой травмой черепа в состоянии шока. Сознание отсутствует, кожа бледная, t0 тела - 35,00С, мышечный тонус понижен, рефлексы отсутствуют, пульс частый и слабый, АД- 50/30 мм рт.ст. В какой клинической стадии шока находится человек?

@ Терминальной

Эректильной

Возбуждения

Торможения

Торпидной

#

159

После травматического удаления зуба больной жалуется на тупую, не имеющую четкой локализации, сильную боль в десне, повышение температуры тела 37,50С. Диагностирован альвеолит. Какой вид боли у данного больного?

@ Протопатическая

Эпикритическая

Висцеральная

Отраженная

Фантомная
160

У больного 37 лет, после отравления неизвестным лекарственным препаратом, отмечаются быстрое непроизвольное стереотипные сокращения мускулатуры лица, имитирующие мигание и прищуривание. К какой форме расстройств двигательной функции нервной системы следует отнести описанное нарушение?

@ Гиперкинез

Гипокинез

Акинезия

Атаксия

-

#

161

Чоловік 48 років поступив в лікарню з загостренням хронічного гломерулонефриту. Обстеження виявило наявність у хворого хронічної недостатності нирок. Чим пояснюється гіперазотемія при хронічній нирковій недостатності?

@ Зниженням клубочкової фільтрації.

Зниженням канальцевої реабсорбції.

Зниженням канальцевої екскреції.

Порушенням білкового обміну.

Порушенням водно-електролітного балансу.

#

162

Хлопчик 12-ти років повернувся зі школи та почав скаржитися на головний біль, нудоту, озноб, періодичний біль у м’язах, втрату апетиту, кволість. Для якого періоду хвороби характерні такі симптоми?

@ Продромального

Латентного

Инкубаційного

Розпалу захворювання

Закінчення хвороби

#

163

З міста аварії доставлено пожежника з ознаками отруєння чадним газом. Який вид гіпоксії для цього притаманний?

@ Гемічна гіпоксія

Ішемічна форма циркуляторної гіпоксії

Застійна форма циркуляторної гіпоксії

Гіпоксична гіпоксія

Дихальна гіпоксія

#

164

У хворого на гіпертонічну хворобу після чергового гіпертонічного кризу стали неможливими довільні рухи правою ногою. Пасивні рухи – в повному об’ємі. При пальпації м’язів правої ноги визначається підвищення їх тонусу. Яка із форм порушення рухової функції має місце у даного хворого?

@ Центральний параліч.

Периферичний парез.

Центральний парез.

Периферичний параліч.

Мозочкова атаксія.

#

165

Кролів годували їжєю з додаванням холестерину. Через 5 місяця виявлені атеросклеротичні зміни в аорті. Назвіть головну причину атерогенеза в даному випадку:

@ Екзогенна гіперхолестеринемія

Переїдання

Гіподінамія

Ендогенна гіперхолестеринемія

-

#

166

У хворого на первинний нефротичний синдром встановлений вміст загального білку крові 40 г/л. Яка причина обумовила гіпопротеїнемію?

@ Протеїнурія

Вихід білка з судин у тканини

Зниження синтезу білка у печінці

Підвищений протеоліз

Порушення всмоктування білка у кішечнику

#

167

Епілептичні судоми у хворого ускладнено розвитком асфіксії, внаслідок руйнування зубних протезів та їх аспірації. Визначте тип недостатності дихання у хворого:

@ Вентиляційна обструктивна

Вентиляційна рестриктивна

Вентиляційна дисрегуляторна

Перфузійна

Дифузійна

#

168

У хворого з серцевою недостатністю виникла аритмія у вигляді генерації позачергових імпульсів в пучку Гіса. Порушення якої функції серцевого м’язу спостерігається в даному випадку?

@ Збудливості

Автоматизму

Провідності

Збудливості та провідності

Скоротливості

#

169

У експериментальних щурів, що тривалий час отримували лише вуглеводну їжу, спостерігалось накопичення води в тканинах. Який патогенетичний механізм є головним у розвитку набряку в даному випадку?

@ Гіпоонкотичний

Мембраногенний

Дисрегуляторний

Лімфогенний

Гіперосмолярний

#

170

У хворого на аденому клубочкової зони кори наднирників (хвороба Конна) спостерігаються артеріальна гіпертензія, напади судом, поліурія. Що є головною ланкою в патогенезі цих порушень?

@ Гіперсекреція альдостерону

Гіпосекреція альдостерону

Гіперсекреція катехоламінів

Гіперсекреція глюкокортикоїдів

Гіпосекреція глюкокортикоїдів

#

171

Хворий 50-ти років страждає на гіпертонічну хворобу. Під час фізичного навантаження у нього з’явились відчуття м’язової слабкості, нестачі повітря, синюшність слизової оболонки губ, шкіри, обличчя, дихання супроводжувалось відчутними на відстані вологими хрипами. Який механізм лежить в основі виникнення такого синдрому?

@ Гостра лівошлуночкова недостатність.

Хронічна правошлуночкова недостатність.

Хронічна лівошлуночкова недостатність.

Колапс.

Тампонада серця.

#

172

Хворому на інсулінзалежний цукровий діабет був введений інсулін. Через деякий час у хворого з "явились слабкість, дратливість, посилення потовиділення. Який основний механізм розвитку гіпоглікемічної коми, що виникла?

@ Вуглеводне голодування головного мозку.

Посилення глікогенолізу.

Посилення кетогенезу.

Посилення ліпогенезу.

Зменшення гліконеогенезу.

#

173

Чоловік віком 30-ти років отримав опромінювання дозою біля 3 Гр. Яка зміна в крові буде через 8 годин після опромінювання?

@ Лімфопенія

Лейкопенія

Гранулоцитопенія

Тромбоцитопенія

Анемія

#

174

У хворого 56-ти років з серцевою недостатністю спостерігається набряк стоп та гомілок, шкіра в місці набряку бліда і холодна. Яка провідна ланка патогенеза набряку в хворого?

@ Підвищення гідростатичного тиску в венулах

Зменшення онкотичного тиску в капілярах

Підвищення проникливості капілярів

Порушення лімфовідтоку

Позитивний водний баланс

#

175

У дитини 5 років через 2 тижня після перенесеної ангіни виник гострий дифузний гломерулонефрит, що характеризувався олігурією, протеїнурією, гематурією, гіперазотемією. Порушення якого процесу у нирках найбільш суттєве для виникнення цих змін?

@ Клубочкова фільтрація

Канальцева реабсорбція

Канальцева секреція

Сечовиведення

Секреція гормонів

#

176

У ліквідатора наслідків аварії на Чорнобильській АЕС під час перебігу гострої променевої хвороби виник геморагічний синдром. Що має найбільше значення в патогенезі цього синдрому?

@ Тромбоцитопенія.

Порушення структури стінки судин.

Підвищення активності факторів фібрінолізу.

Підвищення активноті факторів систем протизсідання крові.

Зменшення активності факторів зсідання крові.

#

177

В експерименті на білого щура подіяли стресовим фактором (електричним струмом) і спостерігали після цього гіпотонію м’язів, артеріальну гіпотензію, гіпотермію, гіпоглікемію. Який період загального адаптаційного синдрому у щура?

@ Фаза шоку

Фаза протишоку

Стадія резистентності

Стадія виснаження

-

#

178

Хворий 43-х років чотири місяця тому переніс травматичну ампутацію лівої нижньої кінцівки. Зараз він скаржиться на відчуття наявності ампутованої кінцівки і постійний сильний, іноді нестерпний біль у ній. Який вид болю в хворого?

@ Фантомний

Каузалгія

Невралгія

Таламічний

Рефлекторний

#

179

У экспериментального животного во втором периоде полного пищевого голодания повышается количество липидов в крови. Какая форма гиперлипемии наблюдается в данном случае?

@ Транспортная

Алиментарная

Ретенционная

Метаболическая

-

#

180

У экспериментального животного удалили мозжечок. В результате движения потеряли плавность, точность, исчезла соразмерность движений. Как называется описанное явление?

@ Атаксия

Гиперкинез

Тремор

Ригидность

Атетоз

#

181

Во время диспансерного осмотра у мужчины 36 лет, по профессии - водителя, уровень АД составил 150/90 мм рт ст. К концу рабочего дня появляется шум в ушах, недомогание, проходящее после отдыха. Диагностирована гипертоническая болезнь. Каков ведущий патогенетический механизм при данной форме заболевания?

@ Нейрогенный

Почечный

Гуморальный

Эндокринный

Рефлексогенный

#

182

У ребенка 14-ти лет, больного дифтерией, в период кризиса, при резком падении температуры, на фоне тахикардии, артериальное давление составляет 70/50 мм рт.ст. К какой форме нарушения сосудистого тонуса относится данное явление?

@ Острая гипотензия

Хроническая гипотензия

Вегето-сосудистая дистония

Гипотоническая болезнь


#

183

После продолжительной и тяжелой болезни у больного снизилось давление (60/40 мм рт.ст.), наблюдается тахикардия, одышка, сознание затемненное. Это состояние можно рассматривать так:

@ Преагонию

Агонию

Шок

Клиническую смерть

-

#

184

У больного желтухой в крови повышено содержание прямого билирубина, желчных кислот, в моче отсутствует стеркобилиноген. При какой желтухе возможно наличие этих признаков?

@ Механической

Печеночной

Паренхиматозной

Гемолитической

Надпеченочной

#

185

У новорожденного ребенка с пилоростенозом наблюдается часто повторяющаяся рвота, сопровождающаяся апатией, слабостью, повышением тонуса мышц, иногда судорогами. Какая форма нарушения кислотно-основного состояния развилась у больного ?

@ Негазовый алкалоз

Газовый алкалоз

Газовый ацидоз

Метаболический ацидоз

Выделительный ацидоз

#

186

У больного через сутки после апендэктомии при анализе крови выявили нейтрофильный лейкоцитоз с регенеративным сдвигом. Какой наиболее вероятный механизмом развития абсолютного лейкоцитоза в периферической крови больного?

@ усиление лейкопоэза

перераспределение лейкоцитов в организме

уменьшение разрушения лейкоцитов

замедление эмиграции лейкоцитов в ткани

активация иммунитета

#

187

У студента после сдачи экзамена в анализе крови обнаружен лейкоцитоз без существенного изменения в лейкоцитарной формуле. Выберите наиболее вероятный механизм развития относительного лейкоцитоза в периферической крови?

@ перераспределение лейкоцитов в организме

усиление лейкопоэза

уменьшение разрушения лейкоцитов

замедление эмиграции лейкоцитов в ткани

ускоренный лейкопоэз

#

188

У больного 19-ти лет обнаружена хроническая приобретенная гемолитическая анемия. Что является ведущим патогенетическим механизмом развития этой патологии?

@ аутоиммунный гемолиз

токсический гемолиз

внутриклеточный гемолиз

гипоосмолярность плазмы

осмотический гемолиз

#

189

Жінка 55 років звернулася зі скаргами на тривалі циклічні маткові кровотечі протягом року, слабкість, запаморочення. При огляді виявлена: блідість шкіри, аналіз крові: Hb – 70 г/л, еритроцити – 3,2*1012/л, колірний показник – 0,6, лeйк – 6,0*109/л, ретикулоцити – 1%. Гіпохромія еритроцитів. Яка анемія у хворої?

@ Хронічна постгеморагічна

Гемолітична

Апластична

В12-фолієводефіцитна

Залізодефіцитна

#

190

Чоловік 56 років потрапив до клініки зі скаргами на загальну слабкість, біль і печіння в області язика, відчуття оніміння в кінцівках. У минулому переніс резекцію кардіального відділу шлунка. В аналізі крові: Hb – 80 г/л; еритроцити – 2,0*1012/л; колірний показник – 1,2 лейкоцити – 3,5*109/л; . Який вид анемії в даного хворого?

@ В12-фолієводефіцитна

Гемолітична

Постгеморагічна

Апластична

Залізодефіцитна

#

191

Хворий після перенесеного епідемічного паротиту почав худнути, постійно відчував спрагу, пив багато води, відмічалось часте сечовиділення, підвищений апетит. Скаржиться на шкірний свербіж, слабкість, фурункульоз. В крові глюкози 16 ммоль/л, кетонові тіла 100 мкмоль/л; глюкозурія. Яке захворювання розвинулось у пацієнта?

@ Інсулінозалежний цукровий діабет

Інсулінонезалежний цукровий діабет

Стероїдний діабет

Нецукровий діабет

Цукровий діабет недостатнього харчування

#

192

Хворий взимку впав у ополонку, замерз на вітрі, захворів. Температура тіла піднялась до 39,70С і коливалась від 39,00С до 39,80С. Назвіть тип температурної кривої в хворого:

@ Febris continua

Febris recurrens

Febris hectica

Febris intermittens

Febris remittens

#

193

Хворому поставлено діагноз: ниркова артеріальна гіпертензія. Назвіть ініціальний патогенетичний фактор розвитку артеріальної гіпертензії в даному випадку?

@ Ішемія нирок

Гіпернатріємія

Гіперальдостеронізм

Збільшення синтезу реніну

Збільшення синтезу ангіотензіну

#

194

При патології нирок в сечі з’являються патологічні складові частини. Поява яких патологічних складових частин сечі свідчить про підвищення проникності клубочкової мембрани?

@ Протеїнурія

Глюкозурія

Аміноацидурія

Алкаптонурія

Піурія

#

195

У хворого на хронічний мієлолейкоз виявлено ознаки анемії – зменшення кількості еритроцитів і вмісту гемоглобіну, оксифільні і поліхроматофільні нормоцити, мікроцити. Який патогенетичний механізм є провідним у розвитку цієї анемії?

@ Заміщення еритроцитарного ростка

Внутрішньосудинний гемоліз еритроцитів

Дефіцит вітаміну В12

Зменшення синтезу еритропоетину

Хронічна кровотрата

#

196

У водолаза, що тривалий час перебував на глибині 40 м, при декомпресії розвинулась кесонна хвороба. Основною патогенетичною ланкою стала емболія:

@ Газова

Повітряна

Жирова

Парадоксальна

Тканинна

#

197

Під час операції на легенях у хворого виникла зупинка серця. Регулярні скорочення його вдалося відновити лише через 10 хв. Найглибші зміни внаслідок гіпоксії сталися в

@ Корі головного мозку

Серці

Печінці

Нирках

Селезінці

#

198

У хворого з облітеруючим ендартеріїтом проведена гангліонарна симпатектомія. Який вид артеріальної гіперемії виник в результаті операції?

@ Нейропаралітична

Нейротонічна

Метаболічна

Робоча

Реактивна

#

199

Жінка 42 років із невралгією трійчастого нерва скаржиться на періодичне почервоніння правої половини обличчя та шиї, відчуття припливу тепла та підвищення шкірної чутливості. Ці явища можна пояснити розвитком артеріальної гіперемії -

@ Нейротонічної

Нейропаралітичної

Метаболічної

Робочої

Реактивної

#

200

Пацієнтка 58-ми років скаржиться на швидку втому, зниження працездатності, сонливість, задишку при швидкій ході. У крові: ер.- 4,0*1012/л, Hb- 92 г/л, КП- 0,6. В мазку крові - велика кількість анулоцитів та мікроцитів. Для якої анемії це характерно?

@ Залізодефіцитної

Постгеморагічної

Гемолітичної

Перніціозної

Серповидноклітинної

#

201

У хворого на гострий міокардит з’явилися клінічні ознаки кардіогенного шоку. Який із вказаних нижче патогенетичних механізмів є провідним в розвитку шоку?

@ Порушення насосної функції серця

Депонування крові в органах

Зниження діастолічного притоку до серця

Зниження судинного тонусу

Збільшення периферичного опору судин

#

202

Під час плевральної пункції було отримано гнійний ексудат. При мікроскопічному дослідженні ексудату знайдено багато гнійних тілець. У результаті руйнування яких клітин крові вони утворюються?

@ Нейтрофіли

Еозинофіли

Базофіли

Лімфоцити

Еритроцити

#

203

Під час розтину тіла померлого від набряку легень у міокарді знайдено велике вогнище жовто-сірого кольору, а в коронарній артерії – свіжий тромб. Який найбільш імовірний діагноз?

@ Інфаркт міокарда

Кардіосклероз

Міокардит

Амілоїдоз

Кардіоміопатія

#

204

У больной 65 лет, страдающей тромбофлебитом глубоких вен голени, в поликлинике, в очереди на прием к врачу, внезапно наступила смерть. На вскрытии трупа в общем стволе и бифуркации легочной артерии найдены свободно лежащие красные рыхлые массы с тусклой гофрированой поверхностью. Какой патологический процесс в легочной артерии нашел патологоанатом?

@ Тромбоэмболию

Тромбоз

Тканевую эмболию

Эмболию инородными телами

Жировую эмболию

#

205

У пацієнта з бронхіальною астмою за дпомогою шкірних алергічних проб встановлено сенсибілізацію алергеном тополиного пуху. Який фактор імунної системи відіграє вирішальну роль в розвитку цього імунопатологічного стану?

@ IgE.

IgD.

IgM.

Сенсибілізовані Т-лімфоцити.


#

206

Ліквідатору наслідків аварії на АЕС, що отримав велику дозу опромінення, проведено трансплантацію кісткового мозку. Через деякий час після проведеної операції у пацієнта діагностовано розвиток реакції трансплантат проти хазяїна. Які антигени послужили пусковим механізмом виникнення цієї реакції?

@ Антигени системи HLA клітин організму ліквідатора.

Антигени системи Rh еритроцитів ліквідатора.

Антигени HBs, HBc, HBE.

Антигенами системи ABO еритроцитів ліквідатора.

Антигени системи HLA клітин організму донора.

#

207

У хворого на запалення трійчастого нерва останніми роками прогресує пародонтит. Який фактор є головним у розвитку пародонтиту?

@ Нейродистрофічні порушення в пародонті

Зменшення активності лейкоцитарної еластази

Послаблення утворення імуноглобулінів

Підвищення тонуса вагуса

Зниження активності калікреїн-кінінової системи

#

208

Пародонтоз уражає у всьому світі до 50% населення віком після 30 років. Вирішальну роль в патогенезі даного захворювання відіграє:

@ Нервово-дистрофічний фактор

Пошкодження калікреїном тканин пародонту

Пошкодження пародонту активними факторами лейкоцитів

Утворення зубного каменю мікрофлорою

Імунні пошкодження тканин
209

Хворому з закритим переломом плечевої кістки накладена гіпсова пов’язка. Наступного дня з’явилася припухлість, синюшність і похолодання кисті травмованої руки. Про який розлад периферичного кровообігу свідчать ці ознаки?

@ Венозна гіперемія

Артеріальна гіперемія

Ішемія

Тромбоз

Емболія

#

210

Через 1-2 доби після видалення у собаки прищитовидних залоз спостерігались: млявість, спрага, різке підвищення нервово-м’язової збудливості з розвитком тетанії. Яке порушення обміну електролітів має місце при цьому?

@ Гіпокальціемія

Гіперкальціемія

Гіпомагніемія

Гіпермагніемія

Гіпонатріемія

#

211

У чоловіка 50 років раптово виникло сильне серцебиття, біль у серці, різка слабкість, підвищення АТ, пульс неправильний з дефіцітом. На ЕКГ виявлено відсутність зубця Р і різні інтервали R-R. Яке порушення серцевого ритму у хворого?

@ Миготлива аритмія.

Дихальна аритмія.

Пароксизмальна тахікардія .

Поперечна блокада серця.

Синусова екстрасистолія.

#

212

При микроскопии мазка экссудата, полученного от крысы с асептическим перитонитом, с добавлением в экссудат птичьих эритроцитов, обнаружены макрофаги, окруженные чужеродными эритроцитами. Какой стадии фагоцитоза соответствует описанная картина?

@ Прилипания

Незавершенного фагоцитоза

Приближения

Погружения

Внутриклеточного переваривания

#

213

У больного обнаружено ожирение, гирсутизм, "лунообразное" лицо, рубцы багрового цвета на коже бедер. АД 180/110 мм рт.ст., глюкоза крови-17,2 ммоль/л . При каком изменении продукции гормонов надпочечников возможна такая картина?

@ Гиперпродукции глюкокортикоидов

Гипопродукции глюкокортикоидов

Гиперпродукции минералокортикоидов

Гипопродукции минералокортикоидов

Гипопродукции адреналина

#

214

У больного на 2-е сутки после развития инфаркта миокарда произошло резкое падение систолического АД до 60 мм.рт.ст. с тахикардией 140 уд/мин, одышкой, потерей сознания. Какой механизм является ведущим в патогенезе развившегося шока?

@ Уменьшение ударного объема крови

Интоксикация продуктами некротического распада

Снижение объёма циркулирующей крови

Пароксизмальная тахикардия

Анафилактической реакции на миокардиальные белки

#

215

Больной 35-ти лет предъявляет жалобы на постоянную жажду, сниженный аппетит. Количество выпиваемой жидкости за стуки 9 л. Суточный диурез увеличен, моча обесцвечена, относительная плотность - 1,005. Наиболее вероятной причиной развития данной патологии у больного является повреждение:

@ гипоталамических ядер

эпителия почечных канальцев

аденогипофиза

эпифиза

базальной мембраны капилляров клубочков

#

216

У пациента с хронической сердечной недостаточностью выявлено увеличение вязкости крови, при капилляроскопии обнаружено повреждение стенок сосудов микроциркуляторного русла. Какое из перечисленных нарушений возможно в данном случае?

@ Сладж-феномен

Тромбоз

Эмболия

Артериальная гиперемия

Венозная гиперемия

#

217

У хворого, 42 років, скарги на болі в епігастральній ділянці, блювоту; блювотні маси кольору “кофейної гущі”; мелена. В анамнезі виразкова хвороба шлунку. Аналіз крові: еритроцити – 2,8*1012/л , лейкоцити – 8*109/л, гемоглобін 90 г/л. Вкажіть найбільш ймовірне ускладнення, яке виникло у хворого?

@ Кровотеча

Пенетрація

Перфорація

Пререродження в рак

Пілоростеноз

#

218

У хворого на цукровий діабет розвинулася діабетична кома внаслідок порушення кислотно-основного стану. Який вид порушення виник при цьому?

@ Метаболічний ацидоз

Метаболічний алкалоз

Респіраторний ацидоз

Газовий алкалоз

Негазовий алкалоз

#

219

У хворого з гломерулонефритом виявлено:анасарка, АТ- 185/105 мм рт.ст., анемія, лейоцитоз, гіперазотемія, гіпопротеінемія. Який показник свідчить про ускладнення гломерулонефриту нефротичним синдромом?

@ Гіпопротеїнемія

Лейкоцитоз

Гіперазотемія

Артеріальна гіпертензія

Анемія

#

220

В ходе клинического обследования пациентки выявлено снижение основного обмена на 40%, увеличение массы тела , снижение температуры тела, одутловатость лица, нарушение половых функций, вялость и апатия, снижение интеллекта. Какое нарушение и какой железы внутренней секреции приводит к появлению данных симптомов?

@ Гипофункция щитовидной железы

Гипофункция паращитовидной железы

Гиперфункция гипофиза

Гипофункция эпифиза

Гиперфункция щитовидной железы

#

221

Пациента на даче ужалила пчела. При осмотре: кисть левой руки горячая, розовая, отечная, в месте укуса пчелы большой красный волдырь. Какой из механизмов является ведущим в развитии отека?

@ повышение проницаемости сосудов

снижение кровенаполнения сосудов

повреждение сосудов при ужалении

снижение онкотического давления в ткани

снижение осмотического давления ткани

#

222

Під час проведення хірургічних маніпуляцій було використано новокаїн з метою знеболення. Через 10хв. у хворого з’явились блідість шкірних покривів, задишка, гіпотензія. Алергічну реакцію якого типу можна запідозрити?

@ Анафілактичного

Цитотоксичного

Імунокомплексного

Стимулюючого

Клітинно-опосередкованого

#

223

У людини на фоні впливу іонізуючого опромінення в крові визначено зменшення кількості гранулоцитів. Чим обумовлений агранулоцитоз?

@ Пригнічення лейкопоезу

Збільшений перехід гранулоцитів в тканини

Розвиток аутоімунного процесу

Підвищене руйнування лейкоцитів

Порушення виходу зрілих лейкоцитів з кісткового мозку

#

224

Пацієнт звернувся зі скаргами на гострий біль у правому підребер’ї. При огляді лікар звернув увагу на пожовтілі склери хворого. Лабораторні аналізи показали підвищену активність АЛТ та негативну реакцію на стеркобілін в калі. Для якого захворювання характерні такі симптоми?

@ Жовчнокам’яна хвороба;

Гемолітична жовтяниця;

Гепатит;

Хронічний коліт;

Хронічний гастрит.

#

225

Вагітна жінка під час пологів втратила близько 800 мл крові. Відмічається тахікардія, артеріальний тиск 100/70 мм рт.ст., тахіпное до 28/хв, Який тип гіпоксії первинно розвивається при такій клінічній ситуації ?

@ Кровяний.

Серцево-судинний.

Змішаний.

Тканевий

Дихальний.

#

226

При проходженні профілактичного огляду у чоловіка, який працює шахтарем, лікар встановив зміни функціонального стану серця, що свідчать про серцеву недостатність в стадії компенсації. Що з нижче перечисленого є головним підтвердженням компенсації діяльності серця?

@ Гіпертрофія міокарда

Тахікардія

Збільшення артеріального тиску

Задишка

Ціаноз

#

227

У хворого на ентерит, що супроводжувався значною діареєю, спостерігається зменшення кількості води в позаклітинному просторі, збільшення її в середині клітин та зниження осмолярності крові. Як називають таке порушення водно-електролітного обміну?

@ Гіпоосмолярна гіпогідратація

Гіперосмолярна гіпогідратація

Осмолярна гіпогідратація

Гіпоосмолярна гіпергідратація

Гіперосмолярна гіпергідратація

#

228

Хвора 65-ти років страждає жовчокам’яною хворобою. Останній час з’явились ознаки ахолічного синдрому внаслідок обтурації жовчних шляхів. Засвоєння яких компонентів їжі буде порушено найбільше?

@ Жирів.

Білків.

Вуглеводів.

Нуклеїнових кислот

Електролітів.

#

229

У хворого на ессенціальну артеріальну гіпертензію розвинувся гіпертонічний криз, що привело до нападу серцевої астми. Який механізм серцевої недостатності є провідним в даному випадку?

@ Перевантаження серця підвищеним опором

Перевантаження серця збільшеним об’ємом крові

Абсолютна коронарна недостатність

Пошкодження міокарда

Порушення надходження крові до серця

#

230

Женщина 43 лет госпитализирована с жалобами на боли в правом подреберьи, кожный зуд. При длительном обследовании установлено: снижение болевой чувствительности и скорости свертывания крови, желтушность кожных покровов, брадикардия, гипотония. Что является наиболее вероятной причиной данного симптомокомплекса?

@ Холемия

Повышение гемолиза эритроцитов

Сахарный диабет

Паренхиматозная желтуха

Печеночная желтуха

#

231

У больного с желтухой в анализе крови обнаружено увеличение общего билирубина за счет непрямой его фракции. Моча и кал интенсивно окрашены. Каков наиболее вероятный механизм указанных нарушений?

@ Повышенный гемолиз эритроцитов

Затруднение оттока желчи из печени

Повреждение паренхимы печени

Нарушение образования прямого билирубина

Нарушение превращения уробилиногена в печени

#

232

Больной на проятжении 5 лет страдет сахарным диабетом, в результате нарушения диеты развилось коматозное состояние. Врач скорой помощи ввел глюкозу. Состояние больного улучшилось. Какая кома наиболее вероятна для данного больного?

@ Гипогликемическая

Ацидотическая

Гипергликемическая

Печеночная

Гипотиреоидная

#

233

Больной проходил очередное обследование в результате которого у него обнаружено гипергликемия, кетонурия, полиурия, глюкозурия. Какая форма нарушения КОС имеет место при наличии этих явлений?

@ Метаболический ацидоз

Газовый ацидоз

Негазовый ацидоз

Газовый алкалоз

Метаболический алкалоз

#

234

После перенесенной стафилакокковой инфекции у больной появился отечный синдром (анасарка), при лабораторном исследовании мочи обнаружена массивная протеинурия. В крови гипопротеинемия, гиперлипемия. Какую патологию можно предположить?

@ Нефротический синдром

Гломелуронефрит

Пиелонефрит

Мочекаменная болезнь

ХПН

#

235

Хворому з метою знеболення, ввели розчин місцевого анестетику. Через декілька хвилин у хворого розвинулась задишка, тахікардія, втрата свідомості. Який шок розвинувся у хворого?

@ Анафілактичний

Кардіогенний

Геморагічний

Травматичний

Опіковий

#

236

У хворого з переломом стегнової кістки в ділянці хірургічної шийки з’явились ознаки гострої правошлуночкової недостатності по причині емболії легеневої артерії. Якого виду емболія зумовила такі симптоми?

@ Жирова

Метастатична

Газова

Повітряна

Тканинна

#

237

Під час операції з приводу гранульоми в ділянці правого верхнього різця виникла кровотеча, яку вдалось зупинити тільки через 3 години. В анамнезі хворого хронічний лімфолейкоз. Що було ймовірно причиною кровотечі?

@ Тромбоцитопенія

Тромбоцитопатія

Лімфоцитоз

Лейкопенія

Еозінофілія

#

238

У альпіністів, які тривалий час знаходились у високогірному районі, спостерігалось збільшення кількості еритроцитів – більше 6*1012/л і гемоглобіну – більше 170 г/л у крові. Який механізм спричинив виникненню цього явища?

@ Посилення продукції еритропоетину нирками.

Посилення гемолізу еритроцитів у кровоносному руслі.

Посилення здатності тканини утілізувати кисень.

Посилення процесів безкисневого збільшення енергії.

Посилення внутрішньоклітинного гемолізу еритроцитів.

#

239

При исследовании мочи у врача-стоматолога, собранной в конце рабочего дня, обнаружено содержание белка 0,7 г/л. В утренней моче таких измениений не обнаружено. Как называется это явление?

@ Функциональная протеинурия

Органическая протеинурия

Неселективная протеинурия

Внепочечня протеинурия

Гематурия

#

240

Жінка 52 р., хвора на рак нижньої щелепи, пройшла курс променевої терапії. Розмір пухлини зменшився. Який з наведених механізмів ушкодження клітини найбільш обумовлює ефективність променевої терапії?

@ Утворення вільних радикалів

Гіпертермія

Лізис NK-клітинами

Тромбоз судин

Мутагене3


Рецензии